Spine Flashcards

1
Q

Anatomy of the Spinal Column

A
  • Descending Tracts (motor)
    • lateral corticospinal tract (LCT)
      • UMN are in the lateral portion of the white matter
      • They synapse with anterior horn cells (ventral) in the grey matter, and more central portion of the spinal cord
    • ventral corticospinal tract
      • Rubriospinal
        • smaller, less axons
        • voluntary muscle control
        • primarily flexion (decorticate)
        • extra-pyramidal
  • Ascending tracts (sensory)
    • Synapse with the doral root ganglion and enter via the posterior horn of the grey matter (doral)
    • dorsal columns
      • deep touch
      • vibration
      • proprioception
    • lateral spinothalamic tract (LST)
      • pain
      • temperature
    • ventral spinothalamic tract (VST)
      • light touch
How well did you know this?
1
Not at all
2
3
4
5
Perfectly
2
Q

ASIA classification of spinal injury

A
How well did you know this?
1
Not at all
2
3
4
5
Perfectly
3
Q

Elderly patient with hyperextension injury and UE weakness. Pathology? Diagnosis? Treatement?

A

<!--StartFragment-->

Central Cord syndrome

  • Pathophysiology
    • hands and upper extremities are located “centrally” in corticospinal tract
    • More research suggests that it is the larger neurons that are affected (the LMN are not affected)
    • Wallerian degeneration may occur below the level of the insult
  • Presentation
    • symptoms
      • weakness with hand dexterity most affected
      • Hyperpathia - burning in distal upper extremity
    • physical exam
      • motor deficit worse in UE than LE
      • hands have more pronounced motor deficit than arms
      • sacral sparing
  • late clinical presentation
    • UE have LMN signs (clumsy)
    • LE has UMN signs (spastic)
  • Imaging
    • Radiographs are always the first initial treatment
      • rule out instability
    • CT should be used to discern any fractures that occurred
    • MRI
      • Look for pre-existing causes for stenosis
      • Edema without haemorrhage on T2 and STIR are commonly found
  • Treatment - nonoperative
    • Initially admit to ICU
    • MAP > 85 - consider vasopressors
    • Immobilization in hard collar
    • Early and intensive PT
    • Monitor for progression of neuro symptoms
  • Operative - controversial; may cause worsening of damage to cord
    • Indications
      • Spinal instability
      • Pre-existing severe stenosis
      • Progressive deficient
    • Timing
      • < 24 hrs if acute instability or if progressive deficiet

<!--EndFragment-->

How well did you know this?
1
Not at all
2
3
4
5
Perfectly
4
Q

Prognosis of central cord syndrome

A

<!--StartFragment-->

  • final outcome
    • good prognosis although full functional recovery rare
    • usually ambulatory at final follow up
    • usually regain bladder control
    • upper extremity and hand recovery is unpredictable and patients often have permanent clumsy hands
  • recovery occurs in typical pattern
    • lower extremity recovers first
    • bowel and bladder function next
    • proximal upper extremity next
    • hand function last to recover
  • Positive predictors
    • young age
    • preinjury employment
    • level of education
    • absence of spinal cord signal abnormality shown by MRI
    • higher initial ASIA motor score
    • absence of spasticity
    • early motor recovery
    • good hand function
  • Poor predictors
    • spinal column instability
    • degree of canal stenosis
    • persistent spasticity
    • medical co-morbidities

<!--EndFragment-->

How well did you know this?
1
Not at all
2
3
4
5
Perfectly
5
Q

Patient with a flexion/compression injury and motor deficiet LE>UE. Diagnosis? Treatment? Outcome?

A

<!--StartFragment-->

Anterior Cord Syndrome

  • Pathophysiology
    • anterior spinal artery injury
      • anterior 2/3 spinal cord supplied by anterior spinal artery
    • direct compression (osseous) of the anterior spinal cord
  • Mechanism
    • usually result of flexion/compression injury
  • Exam
    • LE > UE
      • _​_lateral corticospinal tract
    • dissociated sensory loss
      • _​_lateral spinothalamic (pain, temp)
    • preserved dorsal column
      • DC (proprioception, vibratory sense)
  • Prognosis
    • worst prognosis of incomplete SCI
    • most likely to mimic complete cord syndrome
    • 10-20% chance of motor recovery

<!--EndFragment-->

How well did you know this?
1
Not at all
2
3
4
5
Perfectly
6
Q

Patient was minding his own buisness, and was stabbed in the back with a knife! What spinal cord syndrome would you see? What are the expected findings and prognosis?

A

<!--StartFragment-->

Brown-Segard Syndrome

  • Caused by complete cord hemitransection
    • usually seen with penetrating trauma
  • Exam
    • ipsilateral deficit
      • Lateral corticospinal tract
        • motor function
      • dorsal columns
        • proprioception
        • vibratory sense
    • contralateral deficit
      • Latearl spinothalamic
        • pain
        • temperature
      • spinothalamic tracts cross at spinal cord level (classically 2-levels below)
  • Prognosis
    • excellent prognosis
    • 99% ambulatory at final follow up

<!--EndFragment-->

How well did you know this?
1
Not at all
2
3
4
5
Perfectly
7
Q

What are the ASIA dermatomes and myotomes

A

Random Myotomes

C2 – Flex/Ext C-spine
C3 – Lateral Flexion C-spine
C4 – Shoulder Elevation (shrug)

Random Dermatomes

C2 – Posterolateral Skull
C3 – Just above SC joint and lateral
C4 – Just below AC joint and medial (coracoid)

How well did you know this?
1
Not at all
2
3
4
5
Perfectly
8
Q

What is the ASIA classification of spinal injury?

A

Motor level is the lowest level with > 3 motor

sensory level is the lowest functioning

Can often get some recovery around the zone of injury

How well did you know this?
1
Not at all
2
3
4
5
Perfectly
9
Q

When do you not need c-spine imaging in trauma?

A

low energy trauma

no distracting injuries

no midline tenderness

no neuro symptoms

can rotate head in both directions

How well did you know this?
1
Not at all
2
3
4
5
Perfectly
10
Q

How can you minimize the secondary zone of injury in spinal trauma

A

aggressive fluid resussitation

MAP >85 (prevent hypotensin)

pressors

both in ICU and intra-op

How well did you know this?
1
Not at all
2
3
4
5
Perfectly
11
Q

At what levels do you worry about respiratory and cardiovascular comprimise in SCI?

A
  • Above C3 can have respiratory arrest
    • C3-5 keeps the diaphragm alive
  • Thoracic levels will loose intercostals and so can have respiratory distress
    • intubate before they go into extremis
  • T1-T4 cardiac accelorator fibers
    • don’t function with c-spine injury
  • Injury above T7 - loose your response to stress
    • neurogenic shock; loose the adreniline
    • vasodilation, bradycardia, hypotension
    • treat with pressors, use a swan-ganz catheter to montior fluids
How well did you know this?
1
Not at all
2
3
4
5
Perfectly
12
Q

What is your approach to a patient with a c-spine injury in the emerg and in the OR

A

C-spine precautions

  • c-collar
  • 2 sandbags with head tapped to them
  • off spine board as soon as possible
  • head cut out for kids
  • traction is no longer advocated
  • Intubation/Airway
    • ​to keep precautions
      • ​Manual in line stabilization (MILS)
      • galidoscope
      • blind oral intubation
      • fiberoptic intubation (slow and not recommended for trauma)
      • laryngeal mask airway
      • cricothyrotomy
    • ​above C5 common
    • low threshold
  • ​Circulation
    • ​may have spinal or neurogenic shock (T7)
      • ​fluids, pressors
    • ​hypotension will contribute to secondary injury to spinal cord
      • ​MAP > 85
  • ​Transfers
    • log roll precautions, sliding board
    • turn using spinal positioning table/sandwich
  • Consider neuromonitoring with MEPs
    • very sensitive to inhaled anesthetics
How well did you know this?
1
Not at all
2
3
4
5
Perfectly
13
Q

Complications associated with SCI post-op

A

<!--StartFragment-->

  • Skin problems
    • treatment is prevention
    • start in ER
      • do not leave on back board
      • start log rolling early
    • proper bedding
  • Venous Thromboembolism
    • prevent with immediate DVT prophylaxis
  • Urosepsis
    • common cause of death
    • strict aseptic technique when placing catheter
    • don’t let bladder become overly distended
  • Sinus bradycardia
    • most common cardiac arrhythmia in acute stage following SCI
  • Orthostatic hypotension
    • occurs as a result of lack of sympathetic tone
  • **Autonomic dysreflexia **
    • potentially fatal
    • presents with headache, agitation, hypertension
    • caused by unchecked visceral stimulation
    • check foley
    • disimpact patient
  • Major depressive disorder
    • ~11% of patients with spinal cord injuries suffer from MDD
    • MDD in spinal cord injury patients is highly associated with suicidal ideation in both the acute and chronic phase.

<!--EndFragment-->

How well did you know this?
1
Not at all
2
3
4
5
Perfectly
14
Q

What are the levels associated patient function?

A

<!--StartFragment-->

C1-C3

  • _Ventilator dependen_t with limited talking.
  • Electric wheelchair with head or chin control

C3-C4

  • Initially ventilator dependent, but can become independent
  • Electric wheelchair with head or chin control

C5

- Ventilator independent

  • Has biceps, deltoid, and can flex elbow, but lacks wrist extension and supination needed to feed oneself
  • Independent ADL’s; electric wheelchair with hand control, minimal manual wheelchair function

C6

  • C6 has much better function than C5 due to ability to bring hand to mouth and feed oneself (wrist extension and supination intact)
  • I_ndependent living_; manual wheelchair with sliding board transfers, can drive a car with manual controls

C7

  • Improved triceps strength
  • Daily use of a manual wheelchair with independant transfers

C8-T1

  • Improved hand and finger strength and dexterity
  • Fully independent transfers

T2-T6

  • Normal UE function
  • Improved trunk control
  • Wheelchair-dependent

T7-T12

  • Increased abdominal muscle control
  • Able to perform unsupported seated activities; with extensive bracing walking may be possible

L1-L5

  • Variable LE and B/B function
  • Assit devices and bracing may be needed

S1-S5

  • Various return of B/B and sexual function
  • Walking with minimal or no assistance

<!--EndFragment-->

How well did you know this?
1
Not at all
2
3
4
5
Perfectly
15
Q

What are surgical indications for stabilization in SCI

A

<!--StartFragment-->

  • most incomplete SCI (except GSW)
    • decompress when patient hits neurologic plateau or if worsening neurologically
    • decompression may facilitate nerve root function return at level of injury (may recover 1-2 levels)
  • most complete SCI (except GSW)
    • stabilize spine to facilitate rehab and minimize need for halo or orthosis
    • decompression may facilitate nerve root function return at level of injury (may recover 1-2 levels)
  • consider for tendon transfers
    • e.g. Deltoid to triceps transfer for C5 or C6 SCI
  • metastatic CA patients with > 6 mos life expectancy
  • GSW with
    • progressive neurological deterioration with retained bullet within the spinal canal
    • cauda equina syndrome (considered a peripheral nerve)
    • retained bullet fragment within the thecal sac
    • CSF leads to the breakdown of lead products that may lead to lead poisoning

<!--EndFragment-->

How well did you know this?
1
Not at all
2
3
4
5
Perfectly
16
Q

Diagnosis? Epidemiology? Orthopedic and Non-orthopedic manifestations?

A

Ankylosing Spondylitis

  • An systemic chronic autoimmune spondyloarthropathy characterized by
    • HLA-B27 (90%)
      • carries have a 20-50% risk of having disease
    • RH negative (seronegative)
    • primarily affect axial spine
  • Pathoanatomy - unknown
    • HLA-B27 aggregates with peptides in the joint and leads to a degenerative cascade
      • cytotoxic T-cell autoimmune reaction against HLA-B27
    • enthesitis
      • entheses inflammation leads to bony erosion, surrounding soft-tissue ossification, and eventually joint ankylosis
      • preferentially targets sacroiliac joints, spinal apophyseal joints, symphysis pubis
      • this differentiates from RA, which is a synovial process
    • disc space involvement
      • inflammation of the annulus lead to bridging osteophyte formation (syndesmophytes)
  • Genetics
    • there is a genetic predisposition, but mode of inheritance is unknown
    • HLA-B27 is located on sixth chromosome, B locus
  • Epidemiology
    • 4:1 male:female
    • affects ~0.2% of Caucasian population
    • usually presents in 3rd decade of life
    • juvenile form <16-years-old includes enthesitis
    • f_ewer than 10% of HLA-B27_ positive patients have symptoms of AS
  • Diagnostic criteria
    • bilateral sacroiliitis
    • +/- uveitis
    • HLA-B27 positive
  • Systemic manifestations
    • acute anterior uveitis & iritis
    • heart disease (cardiac conduction abnormalities)
    • pulmonary fibrosis
    • renal amyloidosis
    • ascending aortic conditions (aortitis, stenosis, regurgitation)
    • Klebsilella pneumoniae synovitis
      • HLA-B27 individuals are more susceptible to Klebsilella pneumoniae synovitis
  • Orthopaedic manifestations
    • bilateral sacroiliitis
    • progressive spinal kyphotic deformity
    • cervical spine fractures
      • large-joint arthritis (hip and shoulder)
    • Note: the atlanto-occipital joint is the last to fuse which can lead to atlantoaxial instability
How well did you know this?
1
Not at all
2
3
4
5
Perfectly
17
Q

What are the orthopedic and systemic manifestions of ankylosing spondylitis

A

Systemic manifestations

acute anterior uveitis & iritis
heart disease (cardiac conduction abnormalities)
pulmonary fibrosis
renal amyloidosis
ascending aortic conditions (aortitis, stenosis, regurgitation)
Klebsilella pneumoniae synovitis

Orthopaedic manifestations

bilateral sacroiliitis
progressive spinal kyphotic deformity
cervical spine fractures

large-joint arthritis (hip and shoulder)

Note: the atlanto-occipital joint is the last to fuse which can lead to atlantoaxial instability

How well did you know this?
1
Not at all
2
3
4
5
Perfectly
18
Q

What does this patient have? What does your work-up include?

A

Ankylosing Spondylitis

  • History
    • Full and AMPLE history of pain and function
    • lumbosacral pain and stiffness
      • present in most patients
      • worse in morning
      • insidious onset in 3rd decade of life
    • neck and upper thoracic pain
      • occurs later in life
      • acute neck pain should raise suspicion for fracture
      • Pain will improve as the spine fuses
    • sciatic
      • likely originates from sciatic nerve involvement in the pelvic (piriformis spasm)
    • loss of horizontal gaze
    • Other associated issues including eye irritaiton, chest pain, bowel issues, rashes
      • shortness of breath
    • Neurological symptoms, bowel/bladder
    • PMHx, family medical history, treatment to date
  • Physical exam
    • limitation of chest wall expansion
      • < 2cm of expansion is more specific than HLA-B27 for making diagnosis
    • Schober test
      • used to evaluate lumbar stiffness
    • kyphotic spine deformity
    • chin-on-chest (flexion) deformity of the spine
    • c_hin-brow-to-vertical angle (CBVA)_
      • measured from standing exam of standing lateral radiograph
      • useful for preoperative planning
      • correction of this angle correlates with improved surgical outcomes
    • hip flexion contracture
      • examining patient in supine and sitting position helps differentiate sagittal plane imbalance due to hip flexion contractures or kyphotic spinal deformity
    • sacroiliac provocative tests
      • Faber test
  • Differential (can have similar imaging findings to AS)
    • psoriatic arthritis
    • reactive arthritis
    • arthritis associated with inflammatory bowel disease
    • undifferentiated spondyloarthropathy
  • Radiographs
    • spine - standing full-length AP and lateral of axial spine
      • negative in 50% of cases with spine fractures
      • squaring of vertebrae with vertical or marginal syndesmophytes
      • shining corners” = sclerosis at the disovertebral junction
        • Romanus lesions
      • Zygoapophyseal joints = fusion of posteiror elements
      • late vertebral scalloping (bamboo spine)
      • measurements
        • chin-brow to vertical angle
          • used to measure chin-on-chest deformity
    • pelvis & lower extremity
      • Ferguson pelvic tilt view
        • allows for improved visualization of anterior SI joint
        • xray beam directed 10 to 15 degrees cephalad
        • findings
          • bilateral symmetric sacroiliac erosion
          • earliest radiographic sign is erosion of iliac side of sacroiliac joint
          • joint space narrowing
          • ankylosis
    • ​​If possible to flex-ex views to assess for AAI pre-op to make for safer intubation
    • CT
      • will show bony changes but not active inflammation
      • CT is most sensitive test to diagnose cervical fractures in patients with AS
    • MRI
      • will detect inflammation, making it the best modality for early detection of AS in young patients
      • obtain with cervical fractures to look for epidural hemorrhage
    • Bone scan
      • will show inflammation in the sacroiliac joints, but lacks specificity
  • Labs
    • little diagnostic value
    • often see nonspecific elevations in ESR and CRP
    • RF negative, HLA-B27 positive
  • Diagnostic Injections
    • SI joint injection
      • local anesthetic injected into SI joint under fluoroscopic guidance
      • often most sensitive diagnostic test
How well did you know this?
1
Not at all
2
3
4
5
Perfectly
19
Q

What are the pertient physical exam findings in ank spond

A
  • limitation of chest wall expansion
    • < 2cm of expansion is more specific than HLA-B27 for making diagnosis
  • Schober test
    • used to evaluate lumbar stiffness
  • kyphotic spine deformity
  • chin-on-chest (flexion) deformity of the spine
  • chin-brow-to-vertical angle (CBVA)
    • measured from standing exam of standing lateral radiograph
    • useful for preoperative planning
    • correction of this angle correlates with improved surgical outcomes
  • hip flexion contracture
    • examining patient in supine and sitting position helps differentiate sagittal plane imbalance due to hip flexion contractures or kyphotic spinal deformity
  • Faber test
How well did you know this?
1
Not at all
2
3
4
5
Perfectly
20
Q

What are the radiographic findings you are looking for in ank spond

A
  • spine - standing full-length AP and lateral of axial spine
    • negative in 50% of cases with spine fractures
    • squaring of vertebrae with vertical or marginal syndesmophytes
    • shining corners” = sclerosis at the disovertebral junction
      • Romanus lesions
    • Zygoapophyseal joints = fusion of posteiror elements
    • late vertebral scalloping (bamboo spine)
  • measurements
    • chin-brow to vertical angle
      • used to measure chin-on-chest deformity
  • pelvis & lower extremity
    • Ferguson pelvic tilt view
      • allows for improved visualization of anterior SI joint
      • xray beam directed 10 to 15 degrees cephalad
      • findings
        • bilateral symmetric sacroiliac erosion
        • earliest radiographic sign is erosion of iliac side of sacroiliac joint
        • joint space narrowing
        • ankylosis
  • ​​If possible to flex-ex views to assess for AAI pre-op to make for safer intubation
  • CT
    • will show bony changes but not active inflammation
    • CT is most sensitive test to diagnose cervical fractures in patients with AS
  • MRI
    • will detect inflammation, making it the best modality for early detection of AS in young patients
    • obtain with cervical fractures to look for epidural hemorrhage
  • Bone scan
    • will show inflammation in the sacroiliac joints, but lacks specificity
How well did you know this?
1
Not at all
2
3
4
5
Perfectly
21
Q

What is more specific than HLA-B27 for making diagnosis of ank spond

A

limitation of chest wall expansion

< 2cm of expansion is more specific than HLA-B27 for making diagnosis

How well did you know this?
1
Not at all
2
3
4
5
Perfectly
22
Q

What is the number one procedure performed on patients with ank spond

A

THA

  • Most will recommend do this prior to spinal correction
    • ​if very bad can do osteotomy first to prevent malaligment of the acetabulum
  • Indications
    • ​Unilateral - arthritis
      • ​more verticle, anteverted acetbulum
    • Bilateral - flexion contracture
      • _​_at risk for dislocation
  • Considerations
    • some consider at risk for HO, but no study to say you should put them on prophylaxis
    • uncemented is ok
How well did you know this?
1
Not at all
2
3
4
5
Perfectly
23
Q

What is the earliest sign of sacroilitis in ank spond

A

Ferguson pelvic tilt view

allows for improved visualization of anterior SI joint
xray beam directed 10 to 15 degrees cephalad
findings

bilateral symmetric sacroiliac erosion
earliest radiographic sign is erosion of iliac side of sacroiliac joint

How well did you know this?
1
Not at all
2
3
4
5
Perfectly
24
Q

Diagnosis? Treatment?

A

Kyphotic Deformity from ank spond

  • Rule out hip flexion contracture first
  • Get flex-ex views
    • ​Be aware of potential for atlano-axial instability as atanto-occipital joint is the last to fuse
  • Lumbar osteotomy
    • indications
      • thoracolumbar kyphotic deformity
  • Perioperative preparation
    • Fiberoptic intubation
    • Appropriate positioning to account for kyphosis
    • Neuromonitoring
    • Some advocate for wake up test which is more sensitive than neuromonitoring
  • closing wedge (pedicle subtracting) osteotomy - lowest rates of complications
    • transpedicular decancelization procedure with removal of posterior elements
    • location of osteotomy determined by type of spine flexion deformity
    • hinge located on anterior vertebral body
    • considered procedure of choice due to
      • greater deformity correction (30 t0 40 degrees per level)
      • better fusion and stability due to direct bony apposition
      • fewer complications
  • vertebral body resection
    • entire vertebral body is removed and replaced with a cage
  • single-level opening wedge osteotomy
    • hinges on posterior edge of vertebral body
    • requires rupture of ALL
  • multi-segment opening osteotomy
    • advantage of less bone loss and preservation of ALL by distributing correction over multiple levels
  • outcomes & complications
    • lumbar approach avoids complications of thoracic cage, spinal cord injury, and has potential for greater correction due to long lever arm
    • Dural tears (secondary to dural ectasia)
    • Transient nerve root compression
    • Loss of correction
    • Implant failure
    • Postoperative instability
    • Aortic injury
How well did you know this?
1
Not at all
2
3
4
5
Perfectly
25
Q

Complications associated with ank spond osteotomy

A

Dural tears (secondary to dural ectasia)
Transient nerve root compression
Loss of correction
Implant failure
Postoperative instability
Aortic injury

Note - can get 30-40 deg correction with PSO

How well did you know this?
1
Not at all
2
3
4
5
Perfectly
26
Q

Diagnosis? Treatment?

A

Chin on Chest deformity - Ank Spond

  • Pre-op considerations as previous
  • C7-T1 cervicalthoracic osteotomy
    • Contra-indicated in patients who previously did not have pain, and now have pain as this may indicate a fractre
  • indications
    • cervicothoracic kyphotic (chin-on-chest) deformity
    • goals
      • slight under-correction with f_inal brow-to-chin angle of 10 degrees_
  • advantage of C7-T1 osteotomy include
    • vertebral artery is external to transverse foremen
    • larger canal diameter
    • mobile neural elements
      • requires wide decompression with removal of C7 lateral mass and portions of C7-T1 pedicles to prevent iatrogenic SCI
  • instrumentation
    • usually a combination of lateral mass screws, pedicle screws, and sublaminar hooks
  • postoperative
    • postoperative halo immobilization often required in patients with poor bone quality
  • outcomes & complications
    • increased risk of venous air embolus (VAE) in the sitting operative position
    • C8 palsy
    • Quadrepelegia
    • subluxation
How well did you know this?
1
Not at all
2
3
4
5
Perfectly
27
Q

Diagnosis? Treatment?

A

Fracture Associated with Ank Spond

  • Introduction
    • C7-T1 common becasue of folcrum of head
    • often extension-type fracture that involved all three columns
    • may be occult so if suspicious consider CT scan (best modality to make diagnosis)
    • high mortality rate secondary to epidural hemorrhage
    • 75% neurologic involvement
      • neurologic symptoms often present late
  • traction, orthotic or halo immobilization
    • indications
      • stable spine fractures with no neurologic deficits
    • technique
      • low-weight traction may facilitate reduction
  • spinal decompression with instrumented fusion
    • indications
      • progressive neurologic deficit
      • epidural hematoma with neurologic compromise
      • unstable fracture patterns
    • Positioning
      • Fiberoptic intubation
      • Appropriate positioning to account for kyphosis
      • Neuromonitoring
      • Some advocate for wake up test which is more sensitive than neuromonitoring
    • decompression
      • decision to go anterior or posterior depends on fracture level, presence and location of hematoma, and osteoporosis
    • instrumentation
      • need to obtain long fusion construct
      • multiple points of fixation
        • fosteoporosis
        • long lever arms of the ankylosed spine
    • do not make an effort to correct deformity
  • outcomes & complications
    • progressive deformity
    • nonunion
    • hardware failure
    • infection
How well did you know this?
1
Not at all
2
3
4
5
Perfectly
28
Q

Diagnosis?

A

DISH (Diffuse Skeletal Idiopathic Hyperostosis)

  • Common disorder of unknown etiology characterized by back pain and stiffness
  • Non-marginal syndesmophytes at three successive levels
    • Everywhere in the spine, usually on the right
    • Forestier disease
    • Different from AS
      • No involvement of SI, usually older patients
  • 6-12%
    • More common in older patients with gout and diabetes
  • Sequelae
    • Lumbar stenosis
    • Dysphagia
    • Cervical melopthy
    • Spinal cord injury resulting form even minor trauma
    • Increase risk of heterotopic ossication after THA
  • Presentation
    • Chronic back pain, stiffness
    • Often incidental
  • Imaging
    • Non-marginal syndesmophytes at three successive levels
    • Lateral radiographs can be helpful to differentiae from AS
      • DISH -anterior bone formation, preserved disc space
      • AS - interbody fusion
  • Treatment
    • Usually non-op
    • Surgical treatment may be indicated for certain sequelae
How well did you know this?
1
Not at all
2
3
4
5
Perfectly
29
Q

What does it mean to have a positive provocative discography

A

<!--StartFragment-->

  • **criteria for a positive test **
    • concordant pain response
    • abnormal disc morphology on fluoroscopy and postdiskography CT
    • negative control levels in lumbar spine
  • outcomes - studies show provocative discography is associated with the following
    • increased incidence of lumbar disc herniations
    • loss of disk height
    • endplate changes

<!--EndFragment-->

How well did you know this?
1
Not at all
2
3
4
5
Perfectly
30
Q

Indications for total disc replacement

A

outcomes at least equal to fusion

single level disease

no associated facet OA

How well did you know this?
1
Not at all
2
3
4
5
Perfectly
31
Q

Indications and contra-indications of TDA

A
  • indications
    • controversial
    • most argue single level disc disease with disease-free facet joints is the only true indication
  • Contraindications
    • Radiulopathy
    • Obesity
    • Stenosis
    • Instability
    • Osteoporosis
    • Previous lumbar fusion
    • Infection
    • Fracture
How well did you know this?
1
Not at all
2
3
4
5
Perfectly
32
Q

Outcomes and complications of TDA

A
  • Complications
    • Device migration
    • Vertebral body fractures
    • Persistent low back pain
    • Collapse/subsidence
  • Outcomes
    • showed poor results when lumbar fusion performed for discogenic pain diagnosed with a positive provacative diskography
    • Show to be equivocal outcomes compared to ALIF
      • Lower hositalizaitons
      • fewer complications
How well did you know this?
1
Not at all
2
3
4
5
Perfectly
33
Q

What are poor predictiors of outcomes for arthrodesis and interbody fusion for discogenic pain?

A

<!--StartFragment-->

  • Arthrodesis has not been found to be effective for discogenic pain
  • Associated with poor outcomes of arthrodesis
    • Number of previous surgery
    • WSIB
    • Low household income
    • Older age
    • Litigation
  • Poor predictors of interbody fusion
    • Depression
    • Litigation
    • smoking

<!--EndFragment-->

How well did you know this?
1
Not at all
2
3
4
5
Perfectly
34
Q

What is diabetic amyotrophy

A

<!--StartFragment-->

  • Proximal neuropathy = diabetic amyotrophy
  • Buttock pain radiating down
  • Anterior burning thigh pain associated with weakness
  • Associated with men > 50 with uncontrolled DM2

<!--EndFragment-->

How well did you know this?
1
Not at all
2
3
4
5
Perfectly
35
Q

Differential for lumbar radiculopathy

A

<!--StartFragment-->

  • Hip Arthritis
    • 5-15% older 65
    • People with limp are more likely to have hip pain
    • Can start in buttock, radiate to knee
    • Typically groin pain, aggrevated by activity
    • Will have reduced ROM on exam; OA on imaging
    • Can do an injection to tell the difference
  • Other hip pathology
    • FAI, GT bursitis, Stress fracture
    • Sacral insufficiency fractures, AVN
  • **PVD **<!--StartFragment-->
    • Diabetes, smoking, hyperlipidemia, hypertension
    • Get relief with rest, but is unaffected by flexion
    • Pedal pulses, shiney, hairless legs, erythema
  • Diabetic Amyotrophy
    • Can get distal neuropathy
    • Proximal neuropathy = diabetic amyotrophy
    • Buttock pain radiating down
    • Anterior burning thigh pain associated with weakness
    • Associated with men > 50 with uncontrolled DM2
  • Peripheral parathesias
    • Peroneal nerve compression (ganlion from tib-fib joint)
      • Get EMG if MRI of spine is equivocal from foot drop
      • Usually feel a positive mass, and will have a positive tinels sign
    • LFCN
      • Irritation from obesity, diabetes, pregnancy, tight fitting pants, iatrogenic
    • Piriformis syndrome - from abbert muscle or overuse
      • Buttock pain
      • Aggrevated with sitting
      • Pain over sciatic notch
      • Often have positive SLR
  • Infectious
    • Epidural abcess (often present as neurological deficiet, fevers are second most common)
    • Discitis
    • HIV associated neuropathy,
    • Shingles
    • GBS associated neruopathy, transverse myelitis - full spine MRI to rule out higher lesion and ICU Consult
      • Back pain, weakness, sensory loss
      • Dysthesias, bowel and bladder
  • Iatrogenic
    • Statins = myopathy
    • HIV drug regimins
    • Thalidomide

<!--EndFragment-->

How well did you know this?
1
Not at all
2
3
4
5
Perfectly
36
Q

Positive and negative predictors of disc decompression

A

<!--StartFragment-->

  • positive predictors for good outcome with surgery
    • leg pain is chief complaint
    • positive straight leg raise
    • weakness that correlates with nerve root impingement seen on MRI
    • married status
  • negative predictors for good outcome with surgery
    • worker’s compensation

<!--EndFragment-->

How well did you know this?
1
Not at all
2
3
4
5
Perfectly
37
Q

What is the etiology of failed back syndrome

A
  • Poor patient selection
    • Abnormal psychometrics
    • Chronic pain behavior
    • Unreachable expectations
    • Incorrect diagnosis
  • Wrong procedure
    • Wrong level
    • Missed spinal stenosis
  • Poor technique
    • Battered root syndrome
    • Iatrogenic instability
    • Residual deformity
  • Failure to achieve goal of surgery
    • Pseudarthrosis
    • Incomplete decompression
    • Incomplete correction of deformity
  • Progressive disease
    • Recurrent disk herniation or spinal stenosis
    • Transition syndrome
How well did you know this?
1
Not at all
2
3
4
5
Perfectly
38
Q

What is the most common type of error in spinal surgery? What is the most common cause for failed decompression of stenotic lumbar spine?

A

wrong level

Adjacent level disease

How well did you know this?
1
Not at all
2
3
4
5
Perfectly
39
Q

Indications for discectomy

A

<!--StartFragment-->

  • persistent disabling pain lasting more than 6 weeks
  • failed nonoperative options (and epidural injections)
  • progressive and significant weakness
  • cauda equina syndrome

<!--EndFragment-->

How well did you know this?
1
Not at all
2
3
4
5
Perfectly
40
Q

Indications for MRI in lumbar DDD

A

<!--StartFragment-->

  • Without Gad
    • pain lasting > one month and not responding to nonoperative management or
      • red flags are present
      • infection (IV drug user, h/o of fever and chills)
      • tumor (h/o or cancer)
      • trauma (h/o car accident or fall)
      • cauda equina syndrome (bowel/bladder changes)
  • With gad
    • scar will light up

<!--EndFragment-->

How well did you know this?
1
Not at all
2
3
4
5
Perfectly
41
Q

Indications to decompress a stenotic spine. Indications for fusion?

A

<!--StartFragment-->

  • wide pedicle-to-pedicle decompression
    • indications
      • persistent pain for 3-6 months that has failed to improve with nonoperative management
      • progressive neurologic deficit (weakness or bowel/bladder)
    • outcomes
      • improved pain and function with surgical treatment
      • most common caused of failed surgery is recurrence of disease above or below decompressed level
      • comorbid conditions are strongest predictor of clinical outcomes after decompression for lumbar spinal stenosis
  • wide pedicle-to-pedicle decompression with instrumented fusion
    • indications
      • presence of segmental instability (isthmic spondylolithesis, degenerative spondylolithesis, degenerative scoliosis)
      • surgical instability created by complete laminectomy and/or removal of > 50% of facets <!--StartFragment-->
    • instrumentation is controversial
      • circumferential fusion (with PLIF or TLIF) is accepted but no studies showing its superiority

<!--EndFragment-->

<!--EndFragment-->

How well did you know this?
1
Not at all
2
3
4
5
Perfectly
42
Q

Complications of decompression of lumbar spinal stenosis

A

<!--StartFragment-->

<!--StartFragment-->

  • Complications increase with age, blood loss, and levels fused
    • Major complication
    • wound infection (10%)
    • pneumonia (5%)
    • renal failure (5%)
    • neurologic deficits (2%)
  • Minor complication
    • UTI (34%)
    • anemia requiring transfusion (27%)
    • confusion (27%)
    • dural tear
    • failure for symptoms to improve

<!--EndFragment-->

How well did you know this?
1
Not at all
2
3
4
5
Perfectly
43
Q

Management of a dural tear

A
  • during minimally invasive surgery may not be able to repair it - can keep them bed rest for 48 hours and monitor
  • How to see a tear
    • white pulsitile fluid
    • loss of dural turgor
    • bleeding from previous clean site
  • suction with paddy to control the site; use blunt tools to move nerve roots; ensure adequate hemostasis to see the tear
    • use the microscope or loops to see the tear better
  • Direct repair - 4-0 silk suture
    • If can’t get a repair can use a patch
      • fat patch
      • fascia
      • commerically available
    • when you don’t have to suture
      • minimally invasive surgery
      • no disruption of the arachnoid
      • anterior tear
  • Commericial available, collagen matrix, hydrogels, fibrin glue or sealant (Tseal or Floseal)
    • For all tears
  • Test the repair with valsalva maneuver
  • Close the wound with multilayer watertight closure
  • Lumbar Drains
    • tears you couldn’t close
  • IV antibiotics for 48 hours
  • Bed Rest
    • 48 hours, then attempt stand up test
      • for cervical lumbar tears they should be sitting
    • if continued headache continue bed rest, if continues to fail should consider revision surgery
    • symptomatic treatment of associated symptoms (headache, N/V, pain)
  • Montior closely post-op for successful wound healing and infection
  • Missed leak
    • headache, localized back pain, radiculopathy
    • fluctant mass with drianing would
    • MRI or B2 transferin assay can detect CSF
How well did you know this?
1
Not at all
2
3
4
5
Perfectly
44
Q

Work-up for lumbar spinal pathology

A

<!--StartFragment-->

  • Make sure you rule out other causes of pain and order appropraite imaging - Pelvis/hips, HgA1C, EMG, etc
  • Radiographs
    • standing AP and lateral
      • nonspecific degenerative findings (disk space narrowing, osteophyte formation)
      • degenerative scoliosis
      • degenerative spondylolithesis
    • _flexion/extension _
      • segmental instability and subtle degenerative spondylolisthesis
    • myelography provides dynamic information such as degree of cut off when a patient goes into extension
      • an invasive procedure
  • MRI
    • central stenosis with a thecal sac < 100mm2
    • obliteration of perineural fat and compression of lateral recess or foramen
    • facet and ligamentum hypertrophy
    • MRI findings of spinal stenosis may found in asymptomatic patients
  • CT myelogram
    • more invasive than MRI
    • central and lateral neural element compression
    • bony anomalies
    • bony facet hypertrophy

<!--EndFragment-->

How well did you know this?
1
Not at all
2
3
4
5
Perfectly
45
Q

Epidemiology and pathology of cevical spondylosis and degeneration?

A
  • Age 40-50
  • C5/6 > C6/7 most common
    • levels that have the most extension
  • Risk factors
    • driving
    • smoking
    • lifting
  • Natural aging process of the spine includes spodylosis
    • disc degeneration
      • disc dessication, loss of disc height, disc bulging, and possible disc hernaition
    • joint degeneration
      • uncinate spurring and facet arthrosis
    • ligamentous changes
      • ligamentum flavum thickening and infolding secodary to loss of disc height
    • deformity
      • kyphosis secondary to loss of disc height with resulting transfer of load to the facet and uncovertebral joints, leading to further uncinate spurring and facet arthrosis​​
  • Nerve root compression (radiculopathy) caused by
    • foraminal spondylotic changes
      • secondary to chondrosseous spurs of facet and uncovertebral joints
    • posterolateral disc herniation
      • between posterior edge of uncinate and lateral edge of PL
      • usually affects the nerve root below (C6/7 disease will affect the C7 nerve root)
  • Central cord compression (myelopathy)
    • occurs with canal diameter is < 13mm (normal is 17mm)
    • worse during neck extension whe central cord is pinched between
      • degenerative disc (anterior)
      • hypertrophic facets and infolded ligamentum (posterior)
    • in asians can be caused by ossification of the posterior longitudinal ligament

How well did you know this?
1
Not at all
2
3
4
5
Perfectly
46
Q

What are the differences between the cervical and lumbar spine

A
  • pedicle/nerve root mismatch
    • cervical spine C6 nerve root travels under C5 pedicle (mismatch)
    • lumbar spine L5 nerve root travels under L5 pedicle (match)
    • extra C8 nerve root (no C8 pedicle) allows transition
  • horizontal (cervical) vs. vertical (lumbar) anatomy of nerve root
    • because of vertical anatomy of lumbar nerve root a paracentral and foraminal disc will affect different nerve roots
    • because of horizontal anatomy of cervical nerve root a central and foraminal disc will affect the same nerve root
How well did you know this?
1
Not at all
2
3
4
5
Perfectly
47
Q

Approach to patient with cervical radiulopathy

A
  • History
    • Take a complete and ample history
    • Characterize the pain
      • occipital headache (common)
      • discogenic pain
    • may present with insidious onset of neck pain that is worse with vertebral motion
    • pain in neck, shoulder, or arms
    • Associated neurologic deficiets
      • paresthesias in neck, shoulder, or arms (often global and nondermatomal)
      • weakness or clumsiness in the upper extremity
    • Constitutional symptoms
    • Bowel/bladder/parathesias
    • IV drug use, PMhx
  • Physical exam
    • Examine - cervical spine for alignment
    • Palpate - step-off
    • ROM
    • Spurling Test positive
      • simultaneous extension, rotation, lateral bend, and vertical compression reproduces symptoms in ipsilateral arm
    • shoulder abduction relieves symptoms
      • shoulder abduction (lifting arm above head) often relieves symptoms
      • Differentiates between cervical and shoulder pathology
  • ASIA - common and testable exam findings
  • Radiographs - AP, lateral, oblique, flex-ex
    • common degenerative radiographic findings include
      • degenerative changes of uncovertebral and facet joints
      • osteophyte formation
      • disc space narrowing
      • endplate sclerosis
      • decreased sagittal diameter (cord compression occurs with canal diameter is < 13mm)
      • changes often do not correlate with symptoms
    • 70% of patients by 70 yrs of age will have degenerative changes seen on plain xrays
    • lateral radiograph
      • important to look for sagittal alignment and size of spinal canal
    • oblique radiograph
      • important to look for foraminal stenosis which often caused by uncovertebral joint arthrosis
    • flexion and extension views
      • important to look for angular or translational instability
      • look for compensatory subluxation above or below the spondylotic/stiff segment
  • MRI
    • T2 axial imaging is the modality of choice and gives needed information on the status of the soft tissues. It may show
    • disc degeneration and herniation
    • foraminal stenosis
    • spinal cord changes (myelomalacia)
    • has high rate of false positive (28% greater than 40 will have findings of HNP or foraminal stenosis)
  • CT
    • gives useful information on bony anatomy including osteophyte formation that is compression the neural elements
    • important preoperative planning tool to plan instrumentation
    • study of choice to evaluate for postoperative pseudoarthosis
  • CT myelography
    • indications
      • largely replaced by MRI
      • useful in patients who can not have an MRI due to pacemaker etc
      • useful in patients with prior surgery and hardware causing artifact on MRI
    • technique
      • intrathecal injection of contrast given via C1-C2 puncture and allowed to diffuse caudally
      • lumbar puncture and allowed to diffuse proximally by putting patient in trendelenburg position.
  • Discography
    • controversial and rarely indicated in cervical spondylosis
    • approach is similar to that used with ACDF
    • risks include esophageal puncture and disc infection
  • Nerve conduction studies
  • high false negative rate
  • may be useful to distinguish peripheral from central process (ALS)
    *
How well did you know this?
1
Not at all
2
3
4
5
Perfectly
48
Q

Treatment options for cervical radiulopathy

A
  • Non-operative
    • There are several options, nothing will alter the natural course of the disease
    • PT
    • Hard versus soft collar immobilization - bad if immobilized for too long
    • Cervical root injection - complications
    • Cervical manipulation or traction - no evidence
  • Operative
    • Indications
      • Radicular pain non-responsive to treatment
      • Progressive neurological deficit
    • Option to go anterior or posterior
  • Anterior Cervical Discectomy and Fusion (ACDF)
    • techniques
      • preferred technique and most common
      • uses Smith-Robinson anterior approach
    • anterior plating functions to increase fusion rates and preserve position of interbody cage or strut graft
    • corpectomy and strut graft may be required for multilevel spondylosis
    • single level ACDF is not a contraindication for return to play for athletes
    • complications (see below)
  • Posterior foraminotomy
    • indications
      • foraminal soft disc herniation causing single level radiculopathy
  • Cervical Disc Replacement
    • indications
      • single level disease with minimal arthrosis of the facets
      • studies show equivalence to ACDF
      • effect on adjacent level disease is controversial
How well did you know this?
1
Not at all
2
3
4
5
Perfectly
49
Q

Complications associated with anterior cervical surgery

A
  • post-op hematoma
    • can cause airway comprimise and needs to be urgently decompressed on the ward
  • pseudoarthrosis
    • 5 to 10% for single level fusions, 30% for multilevel fusions
    • if asymptomatic observe
    • if symptomatic treat with either posterior cervical fusion or repeat anterior decompression and plating if patient has symptoms of radiculopathy
      • improved fusion rates but increased complications with posterior fusion
  • recurrent laryngeal nerve injury (1%)
    • laryngeal nerve follows aberrant pathway on the right
    • although theoretically the nerve is at greater risk of injury with a right sided approach, there is no evidence to support a greater incidence of nerve injury with a right sided approach.
    • if you see it, watch over time
      • if not improved over 6 weeks than ENT consult to scope patient and inject teflon
  • hypoglossal nerve injury
    • a recognized complication after surgery in the upper cervical spine with an anterior approach
    • tongue will deviate to side of injury
  • esphogeal rupture/dysphagia
    • dysphagia, choking, aspiration, pain
    • barium swallow and upper endoscopy
    • Keep NPO, broad spectrum antibiotics, consult thoracic surgery
  • vertebral artery injury (can be fatal)
    • alert anesthetist, gain control of the bleed
      • can hemmorhage without control
    • call vascular for direct repair
    • ligation and endovascular procedures are associated with infarct
  • Horner’s syndrome
    • characterized by ptosis, anhydrosis, miosis, enophthalmos and loss of ciliospinal reflex on the affected side of the face
    • caused by injury to sympathetic chain, which sits on the lateral border of the logus colli muscle at C6
  • adjacent segment disease
How well did you know this?
1
Not at all
2
3
4
5
Perfectly
50
Q

How do you treat recurrent laryngeal nerve injury?

A

if you see it, watch over time

if not improved over 6 weeks than ENT consult to scope patient and inject teflon

How well did you know this?
1
Not at all
2
3
4
5
Perfectly
51
Q

Treatment of pseudoarthrosis following ACDF

A
  • if asymptomatic observe
  • if symptomatic treat with either posterior cervical fusion or repeat anterior decompression and plating if patient has symptoms of radiculopathy
  • improved fusion rates but increased complications with posterior fusion
How well did you know this?
1
Not at all
2
3
4
5
Perfectly
52
Q

Compare anterior vs posteior approaches for cervical radiculopathy

A
  • Anterior
    • ​advantages
      • Direct removal of anterior pathology without neural retraction
      • Bone graft restores height and provides indirect foraminal decompression
      • Fusion prevents recurrent neural compression
      • Muscle-sparing approach
    • Disadvantages
      • Fusion-related issues Autograft harvest morbidity Nonunion
      • Plate complications
      • May accelerate adjacent segment degeneration
  • Posterior
    • Advantages
      • Avoids fusion
      • Can be done with minimally invasive techniques
    • Disadvantages__​
      • Symptoms may recur at the surgical segment
      • Removal of anterior pathology would require neural retraction
How well did you know this?
1
Not at all
2
3
4
5
Perfectly
53
Q

Ranawat classficiation of neurological deficiet

A
How well did you know this?
1
Not at all
2
3
4
5
Perfectly
54
Q

Approach to myelopathic patient at initial visit

A
  • History
    • Axial neck pain, occipital headache
    • Nondermatomal numbness, tingling, burning
      • L-hermitte’s sign
      • Upper extremity - weakness and decreased dexterity
      • Lower extremity - gait instability and weakness
    • Bowel and bladder - rare
    • Fever, chills, constitutional symptoms
    • PMHx, vocation, smoker, WSIB
  • Physical Exam
    • Assess spine for obvious deformity, pain step-off
    • Full ASIA with reflexes, UMN and rectal exam
    • Motor - weakness U>L
      • Finger escape - small finger abducts with finger extension due to intrinsic weakness
      • Grip and release - can’t make fist 20 times in 10 seconds
    • Sensory
      • Pinprick
      • Vibratory - severe cases
    • Upper Motor Neuron
      • Hyperreflexia
      • Inverted radial reflex - tapping brachioradialis producses ipsilateral finger flexion
      • Hoffmann’s
      • Clonus (> 3)
      • Babinski
    • Gait and balance
      • Toe-heel walk is difficult
      • Positive Romberg
  • Radiographs
    • general radiographic findings include
      • degenerative changes of uncovertebral and facet joints
      • osteophyte formation
      • disc space narrowing
      • endplate sclerosis
    • lateral radiograph
      • decreased sagittal diameter (cord compression occurs with canal diameter is < 13mm)
      • a Pavlov ratio of less than 0.8 suggest a congenitally narrow spinal canal
      • local kyphosis angle
    • oblique radiograph
      • important to look for foraminal stenosis which often caused by uncovertebral joint arthrosis
    • flexion and extension views
      • important to look for angular or translational instability
      • look for compensatory subluxation above or below the spondylotic/stiff segment
  • MRI
    • MRI is study of choice to evaluate degree of spinal cord and nerve root compression
    • effacement of CSF indicates functional stenosis
    • spinal cord signal changes
      • seen as bright signal on T2 images (myelomalacia)
      • signal changes on T1-weighted images correlate with a poorer prognosis following surgical decompression
    • compression ratio of < 0.4 carries poor prognosis
      • CR = smallest AP diameter of cord / largest transverse diameter of cord
    • has high rate of false positive (28% greater than 40 will have findings of HNP or foraminal stenosis)
  • CT without contrast
    • can provide complementary information with an MRI and is more useful to evaluate OPLL and osteophytes
  • CT myelography
    • more invasive than an MRI but gives excellent information regarding degrees of spinal cord compression
    • particularly useful in patients that can not have an MRI (pacemaker) or has artifact (local hardware)
    • contrast given via C1-C2 puncture and allowed to diffuse caudally or given via a lumbar puncture and allowed to diffuse proximally by putting patient in trendelenburg position.
  • Nerve conduction studies
    • high false negative rate
    • may be useful to distinguish peripheral from central process (ALS)
How well did you know this?
1
Not at all
2
3
4
5
Perfectly
55
Q

What are two measurements to determine canal stenosis on a lateral of the c-spine

A

canal diameter is < 13mm)
a Pavlov ratio of less than 0.8

How well did you know this?
1
Not at all
2
3
4
5
Perfectly
56
Q

How do you calculated compression ratio for the cord in the c-spine on MRI

A

compression ratio of < 0.4 carries poor prognosis

CR = smallest AP diameter of cord / largest transverse diameter of cord

How well did you know this?
1
Not at all
2
3
4
5
Perfectly
57
Q

Compare anterior and posterior approach for cervical myelopathy

A
  • Anterior
    • ​Advantage​s
        1. Direct decompression
        1. Stabilization with arthrodesis
        1. Correction of deformity
        1. Axial lengthening of spinal column
        1. Good axial pain relief
    • Disadvantage
        1. Technically demanding
        1. Graft complications
        1. Need for postoperative bracing
        1. Loss of motion
        1. Adjacent segment degeneration
  • Posterior approach
    • Advantages
        1. Less loss of motion
        1. Not as technically demanding
        1. Less bracing needed
        1. Avoids graft complication
    • Disadvantages__​
        1. Indirect decompression
        1. Preoperative kyphosis and/or instability
      • limitations
        1. Inconsistent axial pain results
        1. Late instability
How well did you know this?
1
Not at all
2
3
4
5
Perfectly
58
Q

Treatment of cervical myelopathy

A

<!--StartFragment-->

  • observation, NSAIDs, therapy, and lifestyle modifications
    • indications
      • mild symptoms
      • patients who are not candidates for surgery
    • modalities
      • medications (NSAIDS, narcotics)
      • immobilization (hard collar in slight flexion)
      • physical therapy for neck strengthening, balance, and gait training
      • traction and chiropractic modalities are not likely to benefit and do have some risks
    • outcomes
      • improved nonoperative outcomes associated with patients with larger transverse area of the spinal cord (>70mm2)
      • studies have shown ~30% improvement with immobilization
  • Prospective studies show improvement in pain, neuro symptoms and function with operative treatment<!--StartFragment-->
    • Considerations
      • number of stenotic levels
      • sagittal alignment of the spine
      • degree of existing motion and desire to maintain
      • medical comorbidities (eg, dysphasia)
  • Unstable spine pre-op
    • Awake fiberoptic intubation, glidoscope
    • C-spine precautions, collar, Sandwich and flip
    • neuromonitoring
  • Anterior Decompression and Fusion (ACDF)
    • indications
      • fixed cervical kyphosis of > 10 degrees
      • < 2 or less disc segments
      • pathology is anterior (OPLL, soft discs, disc osteophyte complexes
    • techniques
      • uses Smith-Robinson anterior approach
      • anterior plating functions to increase fusion rates and preserve position of interbody cage or strut graft
      • corpectomy and strut graft may be required for multilevel spondylosis
      • if > 2 levels
        • 7% to 20% rates of graft dislodgement with cervical corpectomy with associated severe complications including death reported.
    • advantages
      • lower infection rate
      • mild postoperative pain
    • complications & disadvantages
      • dysphagia
      • alteration in speech
      • pseudoarthrosis
        • 12% for single level fusions, 30% for multilevel fusions
        • treat with either posterior wiring or plating or repeat anterior decompression and plating if patient has symptoms of radiculopathy
      • recurrent laryngeal nerve injury (see below)
      • C5 palsy below (see below)
      • esophageal injury
      • airway obstruction (may be due to edema or hematoma)
      • vascular injury
      • vertebral artery injury (can be fatal)
  • Laminectomy with posterior fusion
    • indications
      • multilevel compression
      • kyphosis of < 10 degrees
        • in flexible kyphotic spine, posterior decompression and fusion may be indicated if kyphotic deformity can be corrected
    • contraindications
      • fixed kyphosis of > 10 degrees
  • Laminectomy alone
    • indications
      • rarely indicated due to risk of post laminectomy kyphosis
    • complications
      • progressive kyphosis
      • 11 to 47% incidence if laminectomy performed alone without fusion
      • **Largest predictor of progressive kyphosis is not the number of levels but the inclusion of C7-T1
  • **Laminoplasty **
    • overview & advantages
      • allows for decompression of multilevel stenotic myelopathy without compromising stability and motion (avoids postlaminectomy kyphosis)
      • lower complication rate than multilevel anterior decompression
      • a motion preserving technique
    • indications
      • ossification of PLL
        • has less instability than a multilevel laminectomy
      • Kingwell - lordosis, multi-level
    • technique
      • volume of canal is expanded by hinged-door laminoplasty followed by fusion
      • common techniques include
        • open door (hinge created unilateral at junction of lateral mass and lamina and opening on opposite side)
        • French door (hinge created bilaterally and opening created midline)
      • opening held open by bone, suture anchors, or special plates
    • contraindications
      • cervical kyphosis > 13 degrees
      • servere axial neck pain
        • is a relative contraindication and these patients should be fused
    • complications
      • loss of motion is most common complication
        • associated with a higher degree of postoperative axial neck pain
      • postoperative radiculopathy (see C5 palsy below)
    • outcomes
      • equivalent to multilevel anterior decompression and fusion
  • Combined anterior and posterior surgery
    • indications
      • multilevel stenosis in the rigid kyphotic spine
      • multi-level anterior cervical corpectomies
      • also used to treat a pseudoarthosis following ACDF
      • postlaminectomy kyphosis
  • Anterior Diskectomy and Total Disk Replacements
    • indications
      • only indicated in single level disease due to soft disk herniation

<!--EndFragment-->

<!--EndFragment-->

How well did you know this?
1
Not at all
2
3
4
5
Perfectly
59
Q

Complications associated with cervical myelopathy decompression

A

<!--StartFragment-->

  • **Postoperative C5 palsy **
    • _incidence _
      • reported to occur in ~ 4.6% of patients after surgery for cervical compression myelopathy
      • no significant differences between patients undergoing anterior decompression and fusion and posterior laminoplasty
    • mechanism
      • mechanism is controversial
      • in laminectomy patients it is thought to be caused by tethering of nerve root with dorsal migration of spinal cord following removal of posterior elements
    • prognosis
      • patients with postoperative C5 palsy generally have a good prognosis for functional recovery, but recovery takes time
  • Recurrent laryngeal nerve injury
    • approach
      • in the past it has been postulated that the RLN is more vulnerable to injury on the right due to a more aberrant pathway
      • recent studies have shown there is not an increased injury rate with a right sided approach
    • treatment
      • if you have a postoperative RLN palsy, watch over time
      • if not improved over 6 weeks than ENT consult to scope patient and inject teflon
    • if you are performing revision anterior cervical surgery, and there is an any suspicion of a RLN from the first operation, obtain ENT consult to establish prior injury
    • if patient has prior RLN nerve perform revision surgery on the same as the prior injury/approach to prevent a bilateral RLN injury

<!--EndFragment-->

How well did you know this?
1
Not at all
2
3
4
5
Perfectly
60
Q

Indications to perform ACDF for cervical stenosis

A
  • fixed cervical kyphosis of > 10 degrees
  • < 2 or less disc segments
  • pathology is anterior (OPLL, soft discs, disc osteophyte complexes
How well did you know this?
1
Not at all
2
3
4
5
Perfectly
61
Q

Complications of ACDF

A
  • dysphagia
  • alteration in speech
  • pseudoarthrosis
    • 12% for single level fusions, 30% for multilevel fusions
    • treat with either posterior wiring or plating or repeat anterior decompression and plating if patient has symptoms of radiculopathy
  • recurrent laryngeal nerve injury (see below)
  • C5 palsy below (see below)
  • esophageal injury
  • airway obstruction (may be due to edema or hematoma)
  • vascular injury
  • vertebral artery injury (can be fatal)
How well did you know this?
1
Not at all
2
3
4
5
Perfectly
62
Q

what is the largest predictor of progressive kyphosis following posterior laminectomy

A

**Largest predictor of progressive kyphosis is not the number of levels but the inclusion of C7-T1

How well did you know this?
1
Not at all
2
3
4
5
Perfectly
63
Q

Indications for laminoplasty, contraindications

A
  • Lordosis
  • multiple levels - preserves motion; outcomes are as good as ACDF
  • Contraindications
    • >13 deg kyphosis
    • axial neck pain - indication for fusion
How well did you know this?
1
Not at all
2
3
4
5
Perfectly
64
Q

What are the patterns of instability in a rheumatoid spine?

A
  • atlantoaxial subluxation (most common form of instability)
    • Due to movement and disruption of the ligaments supporting this joint
    • Erosion of the odontoid can be posterior (C1), anterior (transverse), superior
    • Can leave to subluxation in anterior, anterolateral, rotatory
  • basilar invagination
    • Usually results following atlantoaxial subluxation
    • Can lead to sudden death from depression on brainstem and respiratory center
    • Can lead to compression on the anterior vertebral artery leading to TIA or neuro deficiets
  • subaxial subluxation
    • Leads to a characteristic staircase deformity with anterior instability and kyphosis
How well did you know this?
1
Not at all
2
3
4
5
Perfectly
65
Q

Approach to patient with RA C-spine

A

<!--StartFragment-->

  • Symptoms and physcial exam findings similar to cervical myelopathy
  • History
    • Can be asymptomatic
      • neck pain
      • neck stiffness
      • occipital headaches
    • gradual onset of weakness and loss of sensation
    • Can have a feeling of anterior instatiblity, or hear a clicking as the atlanto-axial joint reduces (Sharp-purser test)
    • bowel, bladder, constitutional
    • Meds for RA, previous treatment, other PMHx
  • Physical exam
    • hyperreflexia
    • upper and lower extremity weakness
    • ataxia (gait instability and loss of hand dexterity)
    • Ranawat classificatoin of neurologic impairment
      • ​1 - parasthesia, pain<!--EndFragment--><!--StartFragment-->
      • 2 - subjective weakness, UMN
      • 3 - objective weakness, UMN
        • a - ambulatory
        • b - non ambulatory
  • Radiographs
    • Indications
      • Cervical symptoms > 6 months
      • neurological signs
      • procedure and no imaging 2 years
      • rapid deterioration in function
      • rapid deterioration of carpal and tarsal bones
    • flexion-extension xrays
      • always obtain before elective surgery
      • see subtopic for radiographic lines and measurements
  • CT scan
    • useful to better delineate bony anatomy and for surgical planning
  • MRI
    • study of choice to evaluate degree of spinal cord
    • compression and identify myelomalacia

<!--EndFragment-->

How well did you know this?
1
Not at all
2
3
4
5
Perfectly
66
Q

Indications to get c-spine imaging in a rhematoid patient

A

Cervical symptoms > 6 months
neurological signs
procedure and no imaging 2 years
rapid deterioration in function
rapid deterioration of carpal and tarsal bones

How well did you know this?
1
Not at all
2
3
4
5
Perfectly
67
Q

Indications for surgery in a rheumatoid patient with c-spine symptoms

A
  • progressive neurological deficit
  • pain refractory to medication
  • radiographic risk factors for neurological injury
    • PADI < 14mm with AAI
    • odontoid migration > 5mm above magregor’s line
    • Canal diameter < 14mm in SAS
    • AAI or cord stenosis
    • cervicomedullary angle 135
68
Q

Approach to atlantoaxial instability in a patient with RA

A
  • Introduction
    • present in 50-80% of patients with RA
    • most common to have anterior subluxation of C1 on C2 (can have lateral and posterior)
  • Mechanism
    • caused by pannus formation between dens and ring of C1 that leads to the destruction of transverse ligament and dens
  • Radiographs
    • controlled flexion-extension views to determine ADI and SAC/PADI
    • ADI (atlanto-dens interval)
      • instability defined as > 3.5 mm of motion between flexion and extension views
      • instability alone is not an indication for surgery
      • > 7 mm - alar ligament
      • > 10 mm motion of associated with increased risk of neurologic injury and an indication for surgery
    • PADI / SAC (posterior atlanto-dens interval and space available for cord describe same thing)
      • < 14 mm associated with increased risk of neurologic injury and is an indication for surgery
      • > 13mm is the most important radiographic finding that may predict complete neural recovery after decompressive surgery
  • nonoperative
    • indicated in stable atlantoaxial subluxation
  • posterior C1-C2 fusion
    • indications
      • ADI > 10 mm (on flex-ex views)
      • SAC / PADI < 14 mm
      • progressive instability or pain refractory to non-operative treatment
      • myelopathy
      • progressive neurologic deficits
    • technique
      • Can use posterior wiring (Magerl) or pedicle screws if the joint is reducible
      • adding transarticular screws eliminated need for halo immobilization (obtain preoperative CT to identify location of vertebral arteries)
        • Better if the joint is not reducible
  • occiput-C2 fusion
    • indications
      • atlantoaxial subluxation is combined with basilar invagination
      • resection of C1 posterior arch required for complete decompression
      • leads to indirect decompression of anterior cord compression by pannus
      • may be required if atlantoaxial subluxation is not reducible
  • odontoidectomy
    • indications
      • rarely indicated
      • used as a secondary procedure when there is residual anterior cord compression due to panus formation that fails to resolves with time following a posterior spinal fusion
    • pannus often resolves following posterior fusion alone due to decrease in instability
69
Q

What is the most important radiographic finding that can predict neurological recovery in AAI in RA

A

> 13mm is the most important radiographic finding that may predict complete neural recovery after decompressive surgery

70
Q

Approach to basilar invagination in RA spine

A
  • Introduction
    • also known as superior migration of odontoid (SMO)
      • tip of dens migrates above foramen magnum
    • present in 40%
      • often seen in combination with fixed atlantoaxial subluxation
  • Mechanism
    • cranial migration of dens from erosion and bone loss between occiput and C1&C2
  • Radiographs
    • Ranawat C1-C2 index
      • center of C2 pedicle to a line connecting the anterior and posterior C1 arches
      • normal measurement in men is 17 mm, whereas in women it is 15 mm
      • distance of < 13 mm is consistent with impaction
      • most reproducible measurement
    • McGregor’s line
      • line drawn from the posterior edge of the hard palate to the caudal posterior occiput curve
      • cranial settling is present when the tip of dens is more than 4.5 mm above this line
      • can be difficult when there is dens erosion
    • Chamberlain’s line
      • line from dorsal margin of hard palate->posterior edge of the foramen magnum
      • abnormal if tip of dens > 5 mm proximal Chamberlain’s line
      • normal distance from tip of dens to basion of occiput is 4-5 mm
      • this line is often hard to visualize on standard radiographs
    • McRae’s line
      • defines the opening of the foramen magnum
      • the tip of the dens may protrude slightly above this line (confounded if the dens has erroded)
  • C2 to occiput fusion
    • indications
      • progressive cranial migration (> 5 mm)
      • neurologic compromise
  • transoral or anterior retropharyngeal odontoid resection
    • indications
      • brain stem compromise
71
Q

Approach to subaxial subluxation in RA c-spine

A
  • Introduction
    • present in 20% with RA
    • often occurs at multiple levels
    • often combined with upper c-spine instability
    • lower spine involvement more common with
      • steroid use
      • males
      • seropositive RA
      • nodules present
  • Mechanism
    • soft tissue instability of facet joints and Luschka joints
  • Radiographs
    • subaxial subluxatoin of > 4mm or > 20% of body indicates cord compression
    • cervical height index (body heigh/width) < 2.0 is almost 100% sensitive and specific for predicting neurologic compromise
  • posterior fusion and wiring
    • indications
      • > 4mm subaxial subluxation AND intractable pain and neurologic symptoms
    • Need to extend this to all levels invovled as RA is a progressive disease
72
Q

Complications associated with RA c-spine surgery

A

<!--StartFragment-->

  • High rates of complications in general
  • Infection
  • High mortality rates
  • Failure to improve symptoms
    • outcome less reliable in Ranawat Grade IIIB
  • Pseudoarthrosis
    • 10-20% pseudoarthrosis rate
    • decreased by extension to occiput
  • Adjacent level degeneration

<!--EndFragment-->

73
Q

Pathoantomy of neruological symptoms associated with the following XR

A

Adult Degenerative Spondylolisthesis

  • Pathoanatomy
    • degenerative cascade involves
    • disc degeneration
    • settling of the motion segment
    • buckling of ligamentum flavum
    • microinstability which leads to further degeneration
    • eventual macroinstability and anterolithesis
  • Neurologic symptoms
    • central and lateral recess stenosis
      • a degenerative slip at L4/5 will affect the descending L5 nerve root in the lateral recess
      • caused by slippage, hypertrophy of ligamentum flavum, and encrouchment into the spinal canal of osteophytes from facet arthrosis
    • foraminal stenosis
      • a degenerative slip at L4/5 will affect the L4 nerve root as it is compressed in the foramen
      • vertical foraminal stenosis (loss of height of foramen) caused by
        • loss of disk height
        • osteophytes from posterolateral corner of vertebral body pushing the nerve root up against the inferior surface of the pedicle
      • anteroposterior foraminal stenosis (loss of anterior to posterior area) caused by
        • degenerative changes of the superior articular facet and posterior vertebral body
74
Q

Approach to patient with degenerative scoliosis

A

<!--StartFragment-->

  • Take a complete and ample history
    • mechanical/axial back pain
    • neurogenic claudication
      • defined as buttock and leg pain/discomfort caused by upright walking
      • relieved by sitting
      • not relieved by standing in one place (as is vascular claudication)
    • may be unilateral or bilateral
    • same symptoms found with spinal stenosis
    • cauda equina syndrome (bowel, bladder, saddle parathesias)
    • PMHx, B-Sx
  • Physical exam
    • L4 nerve root involvment (compressed in foramen with L4/5 DS)
      • weakness to quadriceps
        • best seen with sit to stand exam manuever
      • weakness to ankle dorsiflexion (cross over with L5)
        • best seen with heel-walk exam manuever
      • decreased patellar reflex
    • L5 nerve root involvement
      • weakness to ankle dorsiflexion (cross over with L4)
        • best seen with heel-walk exam manuever
      • weakness to EHL
        • provactive walking test
    • have patient walk prolonged distance until onset of buttock and leg pain
      • have patient stop but remain standing upright
        • if pain resolves this is consistent with vascular claudication
      • have patient sit
        • if pain resolves this is consistent with neurogenic claudication (DS)<!--EndFragment--><!--StartFragment-->
  • Radiographs - lumbar AP, lateral neutral/WB, lateral flexion, lateral extension
    • slip evident on lateral xray - myerding grade
    • flexion-extension studies
      • instability defined as 4 mm of translation or 10° of angulation of motion compared to adjacent motion segment
  • MRI
    • best study to evaluate impingement of neural elements
    • T2 weighted sagittal and axials best to look for spinal stenosis
  • CT myelogram
    • helpful in patients in which a MRI is contraindicated (pacemaker)

<!--EndFragment-->

75
Q

How do you define instability in flex-ex views for degenerative spondylolisthesis

A
  • 4 mm of translation
  • 10° of angulation
76
Q

What is the number one reason to consider interbody fusion in DEGENERATIVE SPONDY

A
  • instability (>4mm translation, >10 deg motion on flex-ex)
    • will improve fusion
  • Other benefits:
    • improve saggital alignement
    • indirect decompression
77
Q

Healthy 75yo patient comes in with L5 radiulopathy and progressive symptoms. You get imaging and see the following. What are you options for treatment?

A

Adult Degenerative Spondylolisthesis

  • Nonoperative
    • physical therapy and NSAIDS
    • indications
      • most patients can be treated non-operatively
    • modalities include
      • activity restriction
      • NSAIDS
      • PT - core strengthening
  • epidural steroid injections
    • indications
      • second line of treatment if non-invasive methods fail
  • TLIF for DS is cost effective
    • $42,854 at 2-year follow-up, per QALY
    • <50 000 per QALY is considered cost effective (THA is used as a gold standard)
  • Lumbar decompression alone
    • Renewed interest in recent years
    • Advocates will say there is less morbidity than with fusion
    • Studies show that outcomes are better with fusion
  • lumbar wide decompression with instrumented fusion
    • Age should not be a detourant, good outcomes have been reported
    • indications
      • persistent and incapacitating pain that has failed 6 mos. of non-operative management and epidural steroid injections
      • progressive motor deficit
      • cauda equina syndrome
    • Risk of pseudoarthrosis (36%) without instrumentation does not appear to affect outcomes
    • However, instrumentation is the standard of care
  • Anterior Lumbar Interbody Fusion (ALIF) or PLIF
    • indications
      • reserved for revision cases with pseudoarthrosis
      • Consider PLIF for unstable spine
    • Benefits
      • Enhance fusion rate
      • Improve saggital alignment and disc height
      • Indirect decompression of nerve roots
  • MIS-TLIF
    • May be beneficial in elderly patients with poor bone quality where blood loss and poor bone quality may be an issue
    • Some studies have shown good outcomes, but there have been few complications with traditional methods
    • May be more cost-effective with shorter hospital stay and sooner return to work
78
Q

SPORT Trial findings pertaining to adult DEGENERATIVE SPONDYLOLISTHESIS

A
  • 70% of patients treated surgically had improvement, 20% patients treated non-operatively had improvement
  • No difference in outcomes regarding how patients were fused
  • Iliac bone graft is associated with higher blood loss, longer surgery and no improvement in fusion or reduction in complications
  • Good outcomes with ALIF, but no direct comparision
  • PLIF is better than PLF alone for patients with an unstable lumbar spine (4mm translation, >10 deg motion)
  • Less success with lateral IF
  • Considerations for interbody fusion
    • high risk of non- union and in the presence of local kyphosis
    • high-grade slip
    • symptomatic instability with sagittally oriented facet joints
    • joint effusion on MRI
    • tall intervertebral disk
79
Q

Complications associated with posterior instrumented fusion for adult degenerative spondylolisthesis

A
  • Dural Tear
    • Intra-op
      • Watertight closure
      • 6-0 nonabsorbable suture
      • Commercial sealant
    • 48 hours bed rest with progressive mobilization
    • Post-op can develop psuedomeningocele
      • Headache, subcutaneous fluid collection
      • CT myelogram to assess lesion
  • Pseudoarthrosis
    • 36% with no instrumentation
    • improved with instrumentation
    • growing support for TLIF which may also increase fusion rates and decrease risk of progression
  • Surgical Infection
    • Acutely don’t need to take the hardware out
      • try to treat with abx and take it out once you get a fusion
    • Chronically; get a CT and if there is a fusion you can take the hardware out

<!--EndFragment-->

80
Q

What things on history will tell you the pain from an isthmic spondy is not from disc/facet/OA etc.

A

<!--StartFragment-->

Pain at the level of the defect
History of back pain as an adolescent
Absence of other pathology
Absence of secondary gain
Pathological motion on dynamic radiographs

<!--EndFragment-->

81
Q

Pros and cons of ALIF

A
  • pros
    • may increase chance of union by more complete discectomy and endplate preparation
    • allows improved restoration of disc height
  • cons
    • retrograde ejaculation and sexual dysfunction
    • persistent radiculopathy due to inadequate indirect foraminal decompression
    • persistent low back pain may be caused by nociceptive pain fibers in pars defect that are not removed in an anterior procedure alone
82
Q

Pathology and prognosis of adult isthmic spondylolisthesis

A
  • Defined as spondylolisthesis in an adult caused by a defect in the pars interarticularis (spondylolysis)
  • spondylolysis is seen in 4-6% of population
    • increased prevalence in sports that involve repetitive hyperextension (gymnasts, weightlifters, football linemen)
  • location
    • 82% occur at L5/S1
    • 11% occurs L4/5 (11%)
    • due to forces in the lumbar spine being greatest at these levels and the facet being more coronal
  • Pathophysiology
    • foraminal stenosis
      • adult isthmic spondylolisthesis at L5/S1 often leads to radicular symptoms caused by compression of the exiting L5 nerve root in the L5-S1 foramen (but not by posterior elements as occurs with degenerative)
      • compression can be caused by
        • hypertrophic fibrous repair tissue of the pars defect
        • uncinate spur formation of the posterior L5 body
        • bulging of the L5/S1 disc
    • lateral recess stenosis
      • caused by facet arthrosis and hypertrophic ligamentum flavum
    • central stenosis
      • rare due to fact that these slips are usually only Grade I or II
      • Also rare because the disease process is usually bilateral
      • The posterior elements remain posterior so there is no compression that you would get with a degenerative or congential pathology
  • Prognosis
    • relatively few patients (5%) with spondylolysis with develop spondylolisthesis
    • >10mm occurs in <5% patients
    • slip progression more common in females
    • Progression more common in adults than adolescents
    • More in bilateral
    • L4/5 at higher risk
    • Disk degeneration is associated with risk of slip
83
Q

What clinical factors warn you of higher risk of progression in adult isthmic spondy

A
  • female
  • adult (vs adolsecent)
  • L4/5
  • bilateral pars defect
  • disc degeneration

  • >10mm occurs in 5% of adults*
  • These are also the same factors that increase the chance of the slip being symptomatic*
84
Q

Wiltse-Newman classification

A
85
Q

History and physical for this patient

A
  • History
    • axial back pain
    • most common presentation
    • pain usually has a long history with periodic episodes that vary in intensity and duration
      • Pain can be from the slip or degeneration around the slip - disc, facets
    • Pain from the isthmic fracture
      • Pain at the level of the defect
      • History of back pain as an adolescent
      • Absence of other pathology
      • Absence of secondary gain
      • Pathological motion on dynamic radiographs
    • leg pain - common presentation
      • usually a L5 radiculopathy usually caused by foraminal stenosis at the L5-S1 level
    • Neurogenic symptoms and cauda equina are rare
      • neurogenic claudication
        • caused by spinal stenosis
        • characterized by buttock and leg pain worse with walking
  • Physical exam
    • Examine - gait, lumbar lordosis
    • Feel
      • Stepoff is almost always at L4/5
    • ROM
      • Tight hamstrings
    • Full neuro exam
      • L5 radiculopathy
        • ankle dorsiflexion and EHL weakness
86
Q

Important radiological features of adult isthmic spony

A
  • AP
    • deformity in coronal plane
  • lateral
    • will see spondylolisthesis and pars defect
    • Note dysplastic features of L5
      • Horizonal facets
      • Trapezoidal body
      • Sacral beaking/doming
      • Deficient inferior articular process
    • Note degenerative features - OA, disc space narrowing
  • flexion-extension
    • instability defined as 4 mm of translation or 10° of angulation of motion compared to adjacent motion segment
  • measurements
    • Myerding slip and slip angle are most important
      • Slip angle measures the degree of kyphosis
    • pelvic incidence
      • pelvic incidence = pelvic tilt + sacral slope
      • a line is drawn from the center of the S1 endplate to the center of the femoral head
      • a second line is drawn perpendicular to a line drawn along the S1 endplate, intersecting the point in the center of the S1 endplate
      • the angle between these two lines is the pelvic incidence (see angle X in figure above)
      • correlates with severity of disease
    • pelvic tilt
      • sacral slope = pelvic incidence - pelvic tilt
      • a line is drawn from the center of the S1 endplate to the center of the femoral head
      • a second vertical line (parallel with side margin of radiograph) line is drawn intersecting the center of the femoral head
      • the angle between these two lines is the pelvic tilt (see angle Z in figure above)
    • sacral slope
      • pelvic tilt = pelvic incidence - sacral slope
      • a line is drawn parallel to the S1 endplate
      • a second horizontal line (parallel to the inferior margin of the radiograph) is drawn
      • the angle between these two lines is the sacral slope (see angle Y in the figure above)
  • Correlations with slip
    • Loss of disc height
    • Lumbar index
    • Pelvic incidence > 68.5
  • CT
    • Good to look at the boney anatomy pre-op
    • Can help identify healed fractures for those treated non-op
  • MRI or CT myelography
    • T2 parasagittal images are best study to evaluate for foraminal stenosis and compression of neural elements
  • SPECT is more senstive than MRI or bone scan
87
Q

radiographic correlations with slip in adult isthmic spondy

A

Loss of disc height
Lumbar index < 75
Pelvic incidence > 68.5

88
Q

What is the relationship of pelvic incidence, pelvic tilt and sacral slope

A

PI = PT + SS

Pelvic incidence correlates to severity of disease

89
Q

65 yo healthy female with progressive back pain refractor to PT for 6 months. Treatment options?

A

Adult isthmic spony (L5/S1)

  • oral medications, lifestyle modifications, therapy
    • indications
      • most patients can be treated nonoperatively
    • techniques
      • Flexion based program is better than extension based
        • But evidence is more supportive of surgery than PT
      • activity restriction
      • NSAID
      • role of injections unclear
      • bracing may be beneficial especially in the acute phase
  • L5-S1 decompression and instrumented fusion +/- reduction
    • indications
      • L5-S1 low-grade spondylolisthesis with persistent and incapacitating pain that has failed 6 months of nonoperative management (most common)
        • <50% slip
      • progressive neurologic deficit
      • slip progression
      • cauda equina syndrome
    • Decompression
      • usually involves Gil laminectomy and foraminal decompression
      • removal of loose lamina and scared pars defect allows decompression of nerve root
      • a Gill decompression is destabilizing and should be combined with fusion
    • Instrumentation
      • Improves fusion rates
      • Higher rates of neuro complications associated with screw misplacement
    • interbody fusion (PLIF/TLIF) commonly performed
      • posterior lumbar interbody fusion (PLIF) involves insertion of device medial to facets
      • transforaminal lumbar interbody fusion (TLIF) requires facetectomy and more lateralized and transforaminal approach to the disc space
      • Cons - increased blood loss and longer hospital stay
    • Reduction
      • improved sagittal balance with reduction
      • risk of stretch injury to L5 nerve root with reduction
      • Catastrophic events and 360 fusion has improved with neuromonitoring
  • L4-S1 decompression and instrumented fusion +/- reduction
    • indications
      • L5-S1 high-grade spondylolithesis with persistent and incapacitating pain that has failed 6 months of nonoperative management
        • >50% slip
      • Outcomes of > 50% slip have a higher risk of the following
        • Pseudoarthrosis
        • Progression of slip
        • Neurological damage
        • Implant failure
  • ALIF/PLIT/TLIF
    • indications
      • can be used successfully to treat low-grade isthmic spondylolisthesis even when radicular symptoms are present
      • cannot be used to treat high grade isthmic spondylolisthesis due to translational and angular deformity
    • Benefits
      • Improved fusion rates
      • Indirect decompression
    • outcomes
      • studies have shown good to excellent results in 87-94% at 2 years
      • ALIF alone comparable to PLF
      • PLIF has better saggital alignment with matienence of reduction
        • Does not translate to improved functional outcomes
      • TLIF has been shown to increase fusion and decrease disability scores
      • Complications of PLIF/TLIF
        • Injury to nerve root
        • Incident durotomy
        • Migration of implant
        • Epidural bleeding
90
Q

complications associated with >50% L5/S1 slip

A

Pseudoarthrosis
Progression of slip
Neurological damage
Implant failure

91
Q

Evidence regarding interbody fusion for adult isthmic spondy

A
  • studies have shown good to excellent results in 87-94% at 2 years
  • ALIF alone comparable to PLF
  • PLIF has better saggital alignment with matienence of reduction
    • Does not translate to improved functional outcomes
  • TLIF has been shown to increase fusion and decrease disability scores
  • Complications of PLIF/TLIF
    • Injury to nerve root
    • Incident durotomy
    • Migration of implant
    • Epidural bleeding
92
Q

Risk factors for myelopathy in OPLL

A

congenital stenosis < 60%

OA

increased ROM

<6mm available for cord

lateral deviation of OPLL

93
Q

Risk factors for progression of atlato-axial instabilty in rheumatoid spine

A
  • male
  • positive rheumatoid factor
  • increased CRP
  • subcutaneous nodules
  • aggressive/progressive distal disease
94
Q

What structures make up the posteior ligamentous complex of the spine

A
  • ligamentum flavum
  • interspinous ligament
  • supraspinous ligament
  • facet capsules
95
Q

Measurements indicating normal c-spine alignment

A
  • Lateral view
    • Basion Dens Interval (BDI) <12mm
    • Anterior spinolaminar line < 1mm
    • Drawn from opisthion to C1 arch, should pass C1/C2
    • ADI < 3mm
    • Posterior cortex of atlas should be parallel to anterior cortex of axis
    • SAC > 13mm
  • Open mouth
    • Joint articulations should be <2mm
    • Combine overhang < 7mm
96
Q

What is the Anderson and Montesano classification of occipital condyle fractures?

A
  • Type I
    • comminuted fracture cause by axial load (3% of fractures)
    • cervical orthoses
  • Type II
    • involve extension of basilar skull fracture into condyle (22%)
    • cervical orthoses
  • Type III - avulsion fracture (75%)
    • have a clinical suspicion for occipitocervical dissociation
    • if associated with dissociation treat wtih occipitocervical fusion
97
Q

Classification and treatment of occipitocervical dissocation

A

Craniocervical dissociation

  • severe injury and patients rarely survive
    • most patients die of brainstem destruction
    • identified in 19% fatal cervical injuries
    • in patients that survive very high risk of neurologic injury
    • mechanism can be either translation or distraction
  • Classification
    • Type I - anterior dislocation (most)
    • Type II - longitudinal dislocation
    • Type III - posterior dislocation
  • Harbourview Classification - use this; base on stability
    • Stage 1 - stable
      • MRI evidence of craniocervical osseoligamentous injury, <2 mm displacement with traction test
      • collar for comfort
    • Stage 2 - unstable with stress view
      • Most dangerous because can be unclear
      • MRI evidence of craniocervical osseoligamentous injury, >2 mm displacement with traction test
    • Stage 3 - grossly unstable
      • Craniocervical malalignment of .2 mm on static radiography
  • Stress view
    • Garden-wells tongs
    • start with 5 lbs; monitor for distraction between OA or AA joints
  • Also make sure to screen people for blunt cerebrovascular injuries
  • occipitocervical fusion - posterior segmental stabilization with fusion from Occiput to C2 (minimum)
    • most modern constructs are modular occipital plates that rigidly lock to screw/rod contructs of the subaxial cervical spine
    • the thickest portion of the occiput is located 5 cm lateral to the external occipital protuberance
    • the major dural venous sinuses are located just below the external occipital protuberance and are at risk of penetrative injury during occipitocervical fusion
98
Q

Measurements on plain radiographs to assess occipitocervical dissociation

A
  • low sensitivity to detecting injury (57%)
  • Powers ratio
    • used to diagnosis occipitocervical dislocation
    • technique
      • Powers ratio = C-D/A-B
        • C-D: distance from basion to posterior arch
        • A-B: distance from anterior arch to opisthion
    • significance
      • ratio ~ 1 is normal
      • if > 1.0 concern for anterior dislocation
      • ratio < 1.0 raises concern for
        • posterior atlanto-occipital dislocation
        • odontiod fractures
      • ring of atlas fractures
  • BDI - basin dens interval, BAI - basin axis interval
    • Should be < 12mm
    • Harris lines rule of 12, both should be < 12mm
99
Q

What is the ligamentous anatomy of C1 on C2

A
  • tectorial membrane (most dorsal layer)
    • connects the posterior body of the axis to the anterior foramen magnum and is the cephalad continuation of the PLL
  • cruciate ligaments (middle layer)
    • transverse ligament is the strongest component
      • connects the posterior odontoid to the anterior atlas arch, inserting laterally on bony tubercles.
  • odontoid ligaments including alar and apical ligaments (most ventral layer)
    • paired alar ligaments
      • connect the odontoid to the occipital condyles
      • relatively strong
    • apical ligament
      • runs vertically between the odontoid and foramen magnum.
      • not very strong
100
Q

Imaging suggestive of Atlas fracture

A
  • Radiographs
    • lateral radiographs
      • atlantodens interval (ADI)
        • < 3 mm = normal in adult (< 5mm normal in child)
        • 3-5 mm = injury to transverse ligament with intact alar and apical ligaments
        • > 5 mm = injury to transverse, alar ligament, and tectorial membrane
    • open-mouth odontoid
      • open-mouth odontoid view important to identify atlas fractures
      • sum of lateral mass displacement
        • if sum of lateral mass displacement is > 8.1 mm then a transverse ligament rupture is assured and the injury pattern is considered unstable
        • traditional thinking based on anatomic studies was that a sum of lateral mass displacement of > 6.9 inferred a transverse ligament rupture. But new studies that consider radiographic magnification argue that a transverse ligament rupture should not be inferred unless the lateral mass displacement is > 8.1mm.
  • MRI
    • more sensitive at detecting injury to transverse ligament
  • CT
    • effective at identifying bony fracture patterns such as a Jefferson fx
101
Q

Types of atlas fractures

A
  • Jefferson Fracture
    • bilateral fractures of anterior and posterior arch resulting from axial load
  • lateral mass fracture
  • lamina fracture
  • Transverse ligament injuries
    • Type I - intersubstance tear of transverse ligament
    • Type II - bony avulsion of transverse ligament
102
Q

Treatment of atlas fractures

A
  • halo immobilization for 6-12 weeks (minerva, somi)
    • indications (most atlas fractures) (90% union rates with external immobilization - levigne)
      • intact transverse ligament (< 7mm spread of lateral masses)
      • bony avulsion of transverse ligament (Type II)
    • technique
      • some injuries may require initial traction
    • Risk
      • _​_can lead to distraciton of injury which can risk cock-robin deformity
        • treated with complex osteotomy
      • risk of pseudoarthrosis
      • painful malunion
        • treat with C1-C2 fusion
  • posterior fusion
    • indications
      • midsubstance rupture of transverse ligament (Type I) with widely displaced lateral masses (> 7mm)
    • Options
      • Don’t say these
        • _​_C1-C2
        • C1 alone if ammenable
      • occiput-C2 if C1 comminuted
    • Historically treatment with transverse ligament disruption was treat with occiput-C1 fusion - don’t say this on your oral
    • Now some will fix C1 alone to try to preserve mobility
103
Q

Options for C1-C2 fusion

A
  • C1 lateral mass screws / C2 pedicle screw construct - Harms
    • C1 lateral mass screws
      • distance from ICA is 2mm
    • C2 pedicle screws - still VA s at risk
      • option to use pars or crossed interlaminar screws; more difficult
    • Pros
      • more compatable and can be adjusted as per anatomy
      • less disection
  • C1-2 transarticular screw placement - Magerl
    • is the most stable form of fixation and obviates need for postoperative halo immobilization
    • Contraindicaitons
      • large/medial VA (CT angio)
      • nonanatomic reduction C1/C2
      • hypoplastic C2 pars
      • substatial thoracic kyphosis
    • Optimal screw length is 34mm
  • C1-2 wiring techniques
    • also used but are considered less stable and are usually treated with postoperative halo immobilization. Wiring techniques include
    • Brooke’s technique
    • Gallie’s technique
104
Q

mechanism of injury of traumatic sponylolisthesis

A

<!--StartFragment-->

  • Traumatic anterior spondylolithesis of the axis due to bilateral fracture of pars interarticularis
  • MVA is most common cause
  • Mechanism is
    • hyperextension (leads to fracture of pars)
    • secondary flexion (tears PLL and disc allowing subluxation)
  • Associated injuries
    • 30% have concomitant c-spine fx

<!--EndFragment-->

105
Q

Classification of traumatic sponylolisthesis (hangmans) and treatment options

A
  • Type I
    • Injury (extension and axial loading)
      • < 3mm horizontal displacement
      • No angulation
      • C2/3 disc remains intact
      • stable fx pattern
    • Rigid cervical collar 10 weeks
  • Type IA
    • Injury
      • Atypical, unstable
      • Fracutre threw one bars and out threw body
      • Side-side bending
    • Rigid collar 10 weeks
      • Unpredictable, follow closely
  • Type II
    • Injury (with flexion)
      • > 3mm of horizontal displacement
      • Significant angulation
      • Vertical fracture line
      • unstable fracture pattern
    • Get MRI to discern from TIIA
    • If < 5mm displacement than reduction with traction then halo immobilization for 6-12 weeks
      • assess closely; if you get distraction the injury is a IIA; this you can also use if you don’t have access to MRI
    • If > 5mm displacement consider surgery or prolonged traction.
    • Usually heal despite displacment (autofuse C2 on C3).
  • Type IIA
    • Injury
      • Distraction injury
      • No horizontal displacement
      • Horizontal fracture line
      • Significant angulation
      • Will have injury to disc; unstable in traction
    • Avoid Traction in Type IIA
    • Reduction with hyperextension then halo immobilization for 6-12 weeks.
  • Type III
    • Injury
      • Type I fracture with associated bilateral C2-3 facet dislocation
      • Rare injury pattern
    • Surgical reduction of facet dislocation followed by stabilization required.
106
Q

Options for fusion of traumatic spondylolisthesis of axis fractures

A
  • Most IIA, III
    • some II
  • C1-3 fusion
    • might requrie halo, especially if poor screw purchase across fracture
    • Include C1 if still unstable
  • ACDF (TIIA) C2/3
    • preserves motion
    • more difficult approach
107
Q

What are the components of TLICS

A
  • Injury morphology
    • 1 - compression
      • +1 burst
    • 3 - rotation
    • 4 - distraction
  • Neurological status
    • 2 - nerve root
    • 3 - incomplete SCI
    • 2 - complete SCI
    • 3 - cauda equina
  • Posterior ligamentous complex
    • 2 - suspected
    • 3 - disrupted
108
Q

What are the components of the 3 columns of the spine

A
  • anterior column
    • anterior longitudinal ligament (ALL)
    • anterior 2/3 of vertebral body and annulus
  • middle column
    • posterior longitudinal ligament (PLL)
    • posterior 1/3 of vertebral body and annulus
  • posterior column
    • pedicles
    • lamina
    • facets
    • ligamentum flavum
    • spinous process
    • posterior ligament complex (PLC)
    • instability defined by
109
Q

What makes up the posterior ligamentous complex

A

supraspinous ligament
interspinous ligament
ligamentum flavum
facet capsule

110
Q

Indications for surgical stabilization of a burst fracture

A

injury to the Posterior Ligament Complex (PLC)
progressive kyphosis
> 30°kyphosis (controversial)
> 50% loss of vertebral body height (controversial)
> 50% canal compromise (controversial)

neurological deficiet

Otherwise the best treatment is bracing for pain control

111
Q

Compare anterior vs posterior approach to burst fracture

A
  • Anterior
    • direct decompression
      • better for severe comprimise of canal and comminution
      • better for delay to surgery
    • do not stabilize posterior elements
    • fewer segments to stabilize
    • better correction for deformity >30 deg
  • Posterior
    • easier approach
      • better multiple medical co-mobidities or severe trauma
    • lower non-union
      • more stable fore fracture dislocation
    • indirect decompression
      • ligamentotaxis less effective after 5 days
      • can decompress transpedicular
    • try not to do a laminectomy
    • one level above, one below
112
Q

What are the principles of the anterior thoracolumbar approach

A
  • Side of approach
    • > T8 right (aorta and heart on the left)
      • will require lung deflation
      • can still go left because arteries easier to mobilize than the veins on the right
      • go right to avoid the heart in the thorax
      • Don’t go through the ribs lower than T10
  • Lat dorsi/serattus
  • Enter the rib cage
    • ideally at the level of interest, or one level above
    • intercostal
    • rib resection - better exposure
      • Incise the rib peristeium and perietal layer together to enter the thorax
      • can use the rib as graft
  • ​Costotransversectomy
    • _​_aim for the rib of the same vertebrae (10th rib for T9/10) or if in doubt one rib above
  • Diaphragm is at T12
    • ​leave a 1-2cm cuff to repair to
    • will need to take it down to see T12, L1
  • Will have to ligate semental vessels, but the more you ligate the more you put the cord at risk
  • Artery of Adamkawitz T4-T9
    • _​_if going from the left you should get a pre-op angiography
    • can clamp the vessels and assess evoked potentials before ligating potential vessels
113
Q

Dangers associated with the thoracoabdominal approach

A
  • Intercostal vessels
    • vulnerable during
      • rib resection when running along undersurface of rib, and
      • exposure of vertebrae within chest
    • avoid injury by entering pleura from above the ribs
  • Lungs
    • avoid injury by using sharp instruments wisely when within chest
    • expand lungs every 30 minutes to prevent microatelectasis
  • Esophagus
    • avoid injury through adequate retraction of esophagus while working on spine
  • Artery of Adamkiewicz
    • travels on left side between T9-L2 in 60% of patients
    • must preserve to prevent spinal cord ischemia
114
Q

Dangers associated with the retroperitoneal approach

A
  • Sympathetic chain
    • lateral aspect of vertebral body
  • Genitofemoral nerve
    • anterior surface of psoas muscle attached to fascia
  • Segmental arteries
    • segmental lumbar arteries and veins
  • aorta
  • Ureter
    • lies between psoas fascia and peritoneum
    • attached more firmly to peritoneum
    • stroke to produce peristalsis to confirm
115
Q

Complications associated with anterior thoracolumbar approach

A
  • pulmonary comprimise
  • incisional hernia
  • hardware failure
  • vessel comprimise
  • infection
116
Q

What are the corresponding spinal levels to vascularture of the abdomen

A
  • T12 diaphragm
  • L1 - SMA
  • L2 - renal
  • L4 - common iliac bifurcation
117
Q

Complications associated with anterior approach to the lumbar spine and how to deal with them

A
  • Complications are even higher with revision surgery; always get imaging to assess proximity of anterior structures (vascular/ureters).
  • If doing revision get a senior collegaue or general/thoracic surgery to help
  • Vessel injury
    • left iliac artery most common artery
      • >4mm repair and be aware of compartment syndrome
    • _left common iliac vie_n most common vein
    • Iliolumbar and ascending lumbar vien transverse over L5/S1
    • Gain control of the room - let anaesthesia know, trendelenberg position
    • Pack with epinephrine soaked sponges, gain control with pressure above and below tear (no aggressive use of suction or clamps)
    • Can repair the vien with 5-0 nonabsorbable suture in a figure of 8 fashion
    • If unrepairable can litagate or control bleeding with fibrin sealant, gelfoam, surgicell, floseal
    • Follow closely post-op for DVT
    • If arterial
      • can use clamps above and below
      • should call vascular to repair
      • may need bypass
      • consider fasciotomy, monitor limb with O2 sat and pulses post-op
  • Visceral
    • ureter tear
      • occurs with mobilization of the posas
      • delayed CTA can show ureters pre-op
      • IV methylene blue pre-op
    • Peritoneum
    • Bowel
      • should be repaired immediately
      • can be worse with adhesions or if scarred from chronic inflammation around spine
    • Consult general surgery or urology
    • Pre-op bowel prep can be considered for revision surgery
    • Always have a good bowel regime post-op; ilieus is a very common complication
  • Nervous
    • retrograde ejaculation - presarcal superior hypogastric plexus
      • increased with transperitoneal
    • lumbar sympathetic trunk
  • Nerve root injury
    • over distraction
    • iatrogenic disc hernation
    • posterior protrusion of cage
  • Other
    • Proximal DVT
      • monitor closely
      • DVT prophylaxis
    • arterial thrombosis
  • Increased risk with
    • L4/5 compared to L5/S1
    • anterior osteophyte
    • previous surgery
    • diskitis
    • fracture
118
Q

Injury to what structure causes retorgrade ejaculation

A

presarcal superior hypogastric plexus

119
Q

Risk factors for nonunion of an odontoid fracture

A

> 5 mm displacement (>50% nonunion rate)
fx comminution
angulations > 10 degrees
age > 50 years
delay in treatment

120
Q

Options for treatment of odontoid fracture

A
  • observation alone
    • indications
      • Os odontoideum
      • assuming no neurologic symptoms or instability
  • hard cervical orthosis for 6-12 weeks
    • indications
      • Type I
      • Type II in elderly who are not surgical candidates
        • union is unlikely, however a fibrous union should provide sufficient stability except in the case of major trauma
      • Type III fractures
        • no evidence to support Halo over hard collar
  • halo vest immobilization for 6-12 weeks
    • indications
      • Type II young patient with no risk factors for nonunion
    • contraindications
      • elderly patients
        • do not tolerate halo (may lead to aspiration, pneumonia, and death)
  • Operative
    • posterior C1-C2 fusion
      • indications
        • Type II fractures with risk factors for nonunion
        • Type II/III fracture nonunions
        • Os odontoideum with neurologic deficits or instability
        • Type I with atlantooccipital instability (extremely rare)
    • anterior odontoid osteosynthesis
      • indications
        • Type II fractures with risk factors for nonunion AND
        • acceptable alignment and minimal displacement
        • oblique fracture pattern perpendicular to screw trajectory
          • patient body habitus must allow proper screw trajectory
      • outcomes
        • associated with higher failure rates than posterior C1-2 fusion
  • transoral odontoidectomy
    • indications
      • severe posterior displacement of dens with spinal cord compression and neurologic deficits
121
Q

What are the requirements for performing an anterior odontoid screw? What are the pros and cons?

A
  • acceptable alignment
  • minimal displacement
  • oblique fracture pattern perpendicular to screw trajectory
  • patient body habitus must allow proper screw trajectory
  • Pro - preserves motion segments
  • Con - higher risk of nonunion
122
Q

What are radiographic findings that would suggest atlantoaxial instability in a patient with RA

A
  • ADI (atlanto-dens interval)
    • instability defined as > 3.5 mm of motionbetween flexion and extension views
      • instability alone is not an indication for surgery
    • > 7 mm - alar ligament
    • > 10 mm motion of associated with increased risk of neurologic injury and an indication for surgery
  • PADI / SAC (posterior atlanto-dens interval and space available for cord describe same thing)
    • < 14 mm associated with increased risk of neurologic injury and is an indication for surgery
    • > 13mm is the most important radiographic finding that may predict complete neural recovery after decompressive surgery
123
Q

Common prsentation, location, pathogens and risk factors for discitis?

A
  • Usually seen in adults with a median age for pyogenic osteomyelitis is 50 to 60 years
  • Pathogens include
    • staph aureus is most common (50-65%)
    • staph epidermidis is second most common cause
    • pseudomonas seen in patients with IV drug use (and UTI)
    • salmonella seen in patients with sickle cell disease
    • gram negative infections increasing over last decade and often associated with gram negative infections of the GU and respiratory tract
  • Locations
    • 50-60% of cases occur in lumbar spine
    • 30-40% in thoracic spine
    • ~10% in cervical spine
  • Risk factors include
    • IV drug abuse
    • diabetes
    • obesity
    • malignancy
    • immunodeficiency or immunosuppressive medications
    • malnutrition (serum albumin
    • recent systemic infection (UTI, pneumonia)
    • trauma
    • smoking
124
Q

Pathogenesis of infection in the spine

A
  • Epidural abscess
    • defined as a collection of pus or inflammatory granulation tissue between dura mater and surrounding adipose tissue
    • usually associated with vertebral osteomyelitis
    • present in ~18% of patients with spondylodiskitis
    • 50% of patients with an epidural abscess will have neurologic symptoms
  • Pathogenesis
    • hematogenous seeding
      • generally agreed that inoculation likely occurs through hematogenous seeding (arterial or venous) of the endplates and intervertebral discs
      • endplates contain area of low-flow vascular anastomosis that may provide an environment suited for inoculation
      • involvement of one endplate leads to direct extension into intervertebral discs, followed by direct extension into the second endplate
    • direct inoculation
      • can occur after penetrating trauma, open fractures, and following surgical procedure
      • contiguous spread from local infection
      • most commonly associated with retropharyngeal and retroperitoneal abscesses
  • Neurologic symptoms results from
    • direct infectious involvement of neural elements
    • compression from an epidural abscess
      • epidural abscess present in ~18% of patients with spondylodiskitis
      • 50% of patients with an epidural abscess will have neurologic symptoms
    • neural compression caused by instability of the spine
125
Q

40 yo patient with low back, fevers and chills. Comes in with this XR. Work-up?

A

Assessment for discitis

  • History
    • Take a complete and AMPLE history
    • pain
      • pain is often severe and insidious in onset
      • pain is usually worse with activity and unrelenting in nature
      • pain that awakens patients at night should raise concern for malignancy and infection
    • neurologic symptoms present in 10-20%
      • radiculopathy
      • Myelopathy
    • fever is only present in 1/3 of patients
    • history of UTI, pneumonia, skin infection, of organ transplant are common
    • History of IV Drug use
    • recent travel, history of emmigration
    • PMHx - diabetes, obesity, immunocomprimised malnutrition, trauma, smoking
  • Physical exam
    • Examine (Look)
      • Status of the patient, evidence of IV drug use
      • Obvious deformity of the spine
    • Palpate (feel)
      • obvious stepoff, palpable collection
    • perform careful neurological exam
      • Rectal exam
  • Management
    • ESR, CRP, WBC, blood cultures
      • _​_urine cultures
    • AP, lateral full spine
    • MRI with gad, CT
    • Open biopsy if cultures neg
  • Radiographs
    • findings are usually delayed by weeks
    • paraspinous soft tissue swelling (loss of psoas shadow) seen if first few days
    • disc space narrowing and disc destruction (remember disc destruction is atypical of neoplasm) seen at 7-10 days
    • endplate erosion or sclerosis seen at 10-21 days
    • osteopenia
  • CT
    • usefull to show bony abnormalities, abscess formation, and extent of bony involvement
    • CT myelogram is contraindicated
  • MRI
    • MRI with gadolinium contrast most sensitive (96%) and specific (93%) imaging modality for diagnosis of spinal osteomyelitis
    • also most specific imaging modality to differentiate from tumor
    • findings include
      • paraspinal and epidural inflammation
      • disc and endplate enhancement with gadolinium***
      • Destroys endplate and then goes up into the body
      • T2-weighted hyperintensity of the disk and endplate
  • Bone scan
    • Technetium Tc99m bone scan are 90% sensitive but lacks specificity
  • combined Technetium Tc99m and gallium 67 scan is both more specific and more sensitive than Technetium Tc99m alone
  • indium 111 labeled scan not recommended due to poor sensitivity (17%)
126
Q

40 yo with low back pain comes in with fevers, chills, low back pain. You find out he is an IV drug user. BW is elevated, blood and urine cultures are negative but CT guided biopsy grows pseudomonas. Management?

A

Discitis

  • Consult ID
  • Goals of treatment
    • identify organism
      • Blood cultures
      • CT guided biposy
      • Open biopsy
    • eliminate infection
      • Directed antibiotics
    • prevent or improve neurologic deficits
    • maintain spinal stability
    • Provide adequate nutrition
  • Confirm Organism
    • blood cultures
      • 33% (reports show 25%-66%) of patients with spondylodiskitis have positive blood cultures which are 85% accurate for isolating the correct organism
      • blood culture yield is improved by withholding antibiotic and obtaining cultures when patient is febrile
    • CT guided biopsy
      • indicated in patients who do not have indications for immediate open surgery and blood cultures are negative
      • can provide diagnosis in 68-86% of patients
      • can be guided by fluoroscopy or by CT scan
    • cultures should be held for 10 days and sent for:
      • aerobic
      • anaerobic
      • fungal
      • acid-fast cultures
    • open biopsy
      • indicated when tissue/organism diagnosis can not be made with noninvasive techniques
      • anterior, costotransversectomy, or transpedicular approach used
  • bracing and long term antibiotic (6-12 weeks)
    • indicated in most cases and successful in 80%
    • bracing
      • helps improve pain and prevent deformity
      • rigid cervicothoracic orthosis or halo required for cervical osteomyelitis
    • antibiotics
      • if patient is septic or critically ill that start antibiotics immediately, otherwise hold until organism identified
      • usually treated with IV antibiotics until signs of improvement (~ 6 weeks) and then converted to PO antibiotics
      • use broad spectrum antibiotics until organism is identified
      • vancomycin for pencicillin-resistent and gram-positive bacteria
      • third-generation cephalosporin for gram-negative coverage
      • new antibiotic-resistant strains of microorganisms are becoming more common and failure to diagnose can have dire consequences organisms include
        • MRSA (methicillin-resistent Staph aureus)
        • VRSA (vancomycin resistant Staph aureus)
        • VRE (vancomycin resistant enterococcus)
      • newer generation antibiotics for antibiotic resistant organisms include linezolid and daptomycin
  • anterior debridement and strut grafting, +/- posterior instrumentation
    • indications
      • abcess
      • neurologic deficits (for any reason)
      • progressive deformity
      • gross spinal instability
      • persistent infection despite antibiotic (BW still elevated)
    • Approach
      • Retroperitoneal
      • Thoracolumbar
        • Lateral decubitus, pad all pressure points
          • T2-9 right
          • T10-L2 left
        • 2 rbis above level, divide lat dorsi and serratous anterior
        • Intercostal vs rib resection (5th or 6th intercostal space)
        • Enter pleural, deflate the lung, retract esophagus, ligate intercostal vessesl
    • strut graft selection
      • autogenous tricortical iliac crest, rib, or fibula strut graft have proven safe and effective in presence of acute infection
      • allograft being used with good results, but autogenous sources theoretically have better incorporation
      • a recent study showed improved deformity correction with titanium mesh cages filled with autograft (followed by posterior instrumentation)
    • instrumentation
      • spinal instrumentation in presence of active infection is controversial
      • some advocated I&D followed by staged instrumentation
      • some advocate a single procedure with bone graft and instrumentation in the presence of an active infection
      • titanium is preferred over stainless steel
  • posterior instumentation
    • Indications
      • severe kyphotic deformity
      • multilevel anterior construct
    • posterior instrumentation can be performed at same time or as a staged procedure
127
Q

Indications for surgery in diskitis

A
  • Anterior debridement with strut graft
    • abcess
    • neurologic deficits (for any reason)
    • progressive deformity
    • gross spinal instability
    • persistent infection despite antibiotic (BW still elevated)
  • Posterior instrumentation
    • severe kyphotic deformity
    • multilevel anterior construct
128
Q

Goals of treatment in diskitis

A
  • Identify organism
    • Blood/urine cultures
    • CT guided biposy
    • Open biopsy
  • Eliminate infection
    • Directed antibiotics
  • prevent or improve neurologic deficits
  • maintain spinal stability
  • Provide adequate nutrition
129
Q

65 yo male with diabetes comes in with progressive neurological deficiet, fevers and this imaging? How do you work him up?

A

Posterior Epidural Abcess

  • adults > 60 years of age
  • vertebral osteomyelitis and diskitis (spondylodiscitis)
  • present in ~18% of patients with spondylodiskitis
  • Neurologic deficits
    • 33% of patients with an epidural abscess will have neurologic symptoms
    • 4-22% incidence of permanent paralysis
    • can be caused from direct compression or infarction of spinal cord blood flow
  • Location
    • usually dorsal in thoracolumbar spine
  • Risk factors include
    • IV drug abuse
    • immunodeficiency
    • malignancy
    • HIV
    • immunosuppressive medications
    • recent spinal procedure
  • Outcome
    • preoperative degree of neurologic deficits is most important indicator of clinical outcome
    • mortality ~ 5%
  • History
    • Take a complete and AMPLE history
    • Characterize the pain - pain is often severe and insidious in onset an occurs in 87%
    • Associated numbness, weakness; difficulty with gait or hands
    • Constitutional symptoms
    • systemic illness more profound than patients with vertebral osteomyelitis
    • fever present in ~50%
    • Bowel/bladder/parasthesias
    • IV drug use, PMHx, recent travel/infectious contacts
  • Physical exam
    • Examine - patient, deformity
    • Palpate - step-off
    • ROM/ASIA
    • neurologic deficits present in ~33%
    • may present as a radiculopathy or a myelopathy
  • Management
    • ESR, CRP, WBC, Blood cultures, urine cultures
      • WBC elevated 42% of time
      • ESR/CRP elevated 90% of time
    • AP lateral spine, MRI with gad
  • Radiographs - AP/Lateral
    • Often normal
  • MRI is the study of choice
    • MRI with gadolinium is the imaging modality of choice for diagnosis of spinal epidural abscess
    • shows extent of abscess, presence of vertebral osteomyelitis, and allows evaluation of neurologic compression
    • gadolinium allows differentiation of pus from CSF
    • a ring enhancing lesion is pathognomonic for abscess
  • CT
    • poor sensitivity for epidural abscess
  • CT myelogram
    • 90% sensitivity but invasive
130
Q

What is the most important clinical outcome with spinal epidural abcess

A

preoperative degree of neurologic deficits is most important indicator of clinical outcome

131
Q

Diagnosis? Treatment?

A

Posterior Epidural Abcess

  • Goals
    • early diagnosis is most essential factor in preventing devastating outcomes
    • historically presence of epidural abscess has been considered a medical surgical emergency
    • there has been a recent trend towards nonoperative management as new studies shows nonoperative treatment effective in patients without neurologic deficit
  • bracing and long term antibiotic
    • indicated only patients with
      • no neurologic deficits
      • small abscess
      • patient capable of close clinical followup
      • those who are not candidates for surgery due to medical comorbidities
  • surgical decompression is indicated in most patients with spinal epidural abscess.
    • indications for surgery include
      • neurologic deficits
      • persistent infection
      • progressive deformity
      • gross spinal instability
    • technique depends on location of abscess and extent of infection (presence of spondylodiscitis)
  • decompressive laminectomy
    • most common form of operative treatment
    • indicated when abscess is posterior and there is no contiguous spondylodiscitis
    • avoid wide decompression and facetectomy as it will result in spinal instability
  • anterior debridement and strut grafting
    • indications
      • abscess is located anteriorly
      • anterior vertebral body and discs are involved (presence of spondylodiscitis)
    • technique
      • see vertebral osteomyelitis treatment for technique details
  • postoperative antibiotics
    • indicated for 2-4 weeks if no bony involvement of infection
    • indicated for 6 weeks if bony involvement
132
Q

Indications for the use of antibiotics alone for epidural abcess?

A
  • no neurologic deficits
  • small abscess
  • patient capable of close clinical followup
  • those who are not candidates for surgery due to medical comorbidities
133
Q

How long to give antibiotics post-op epidural spinal abcess

A
  • indicated for 2-4 weeks if no bony involvement of infection
  • indicated for 6 weeks if bony involvement
134
Q

This patient comes in with back pain. Immigrated from china 2 years ago. What’s your approach?

A

Pott’s disease - spinal TB

  • History
    • Complete and AMPLE history
    • Characterize the onset of the pain, history of TB/travel/contacts/immigration
    • onset of symptoms of tuberculous spondylitis is typically more insidious than pyogenic infection
    • constitutional symptoms
      • chronic illness
      • malaise
      • night sweats
      • weigh loss
      • back pain
      • Bowel/bladder parasthesias
    • presents and appears more similar to cancer than infection
  • Physical exam
    • Examine (Look)
      • Deformity
    • Palpate (Feel)
      • Areas of pain, stepoff
    • ROM (Move)
      • Full ASIA exam
      • neurologic deficits (present in 10-47% of patients with Pott’s Disease)
  • Management
    • ESR, CRP, WBC
      • ESR up in 25%
    • CXR - 66% will be abnormal
    • full spine XR, CT, MRI with gad
    • ID Consult
  • Spine radiographs
    • early infection
      • shows involvement of anterior vertebral body with sparring of the disc space (this finding can differentiate from pyogenic infection)
    • late infection
      • shows disk space destuction, lucency and compression of adjacent vertebral bodies, and development of severe kyphosis
  • MRI with gadolinium contrast
    • remains preferred imaging study for diagnosis and treatment
  • Nucleur medicine studies (combination of technetium and gallium)
    • shown to have highest sensitivity for detecting infection
  • CT guided biopsy
    • PPD (purified protein derivative of tuberculin)
      • positive in ~ 80%
    • ​PCR can speed up culture results
    • AFB take 10 weeks to grow
135
Q

Emmigrant from indonesia with back pain. CT guided biopsy positive for AFB. Treatment?

A

TB - Pott’s Disease

  • Diagnosis
    • CT guided biopsy with cultures and staining effective at obaining diagnosis
    • mycobacteria (acid-fast bacill) may take 10 weeks to grow in culture
    • PCR allows for faster identification (95% sensitivity and 93% accuracy)
  • chemotherapy is the mainstay of treatment
    • treated with isoniazid, rifampin, and pyrazanamide for 9 to 18 months
    • ethambutol and streptomycin added for part of treatment
    • Follow with serial imaging for progression of kyphosis
    • spinal orthosis
    • may be used for pain control and prevention of deformity
  • Radical debridement with stabilization
    • unstable when all three columns are invovled
      • two with trauma because acute and the inflammatory tissue provides support
    • Begin chemotherapy 10-12 days prior
    • indications
      • neurologic deficit
      • spinal instability or progressive kyphosis
      • advanced disease with caseation preventing access by antibiotics
      • failure of nonoperative treatment after 3 to 6 months
    • advantages of surgical treatement
      • less progressive kyphosis
      • earlier healing
      • decreased sinus formation
      • in patients with neurologic deficits, early debridement and decompression led to improved neurologic recovery
    • Anteiror approach with strut graft
      • short segment disease with moderate kyphosis
      • lateral decubitus with anterior approach
      • autogenous and allograft strut grafts are acceptable with good results
        • rib strut grafts are readily available in thoracic approach
        • iliac crest grafts another option with mesh
      • Anterior plating with one level above and one level below or rod construct
      • good outcomes with anterior approach alone, but if more severe or long segement might want to also go posterior
      • Posterior stabilization means more surgery, more difficult nursing and has been shown not to be necessary
    • Posterior approach with transpedicular decompression +/- pedical subtraction osteotomy
      • >5cm disease with >30 deg kyphosis
      • better correction, more stable construct but technically very difficult
      • better correction for more chronic situation
136
Q

Risk factors and associations with post-op spinal infections

A
  • Hard to diagnose, difficult to treat
  • Increased with
    • Trauma
    • ICU, multiple co-mobidities, malnutrition
    • Instrumentation
    • Microscope
    • Complete neuro deficit
    • Cognitive impairment
    • Posterior approach
    • Anterior approach carries low infection
  • Risk factors
    • Obesity
    • Diabetes
    • long-term steroid use
    • alcohol abuse
    • prior infection
    • prior spinal surgery
    • preoperative hospitalization >1 week
    • Surgery > 3 hours
    • Blood loss > 1L
  • Prevention
    • Periop antibiotics are the most important thing
    • Release retractors, debride necrosis, don’t use drains
    • Negative pressure room
137
Q

How do post-op spinal wound infections typical present

A
  • Superficial Wound
    • Will present like any other wound with erythema, pain, redness
  • Deep Infection
    • Concerning with purulent drainage, increaseing pain
    • Fever >39 is in the number one predictor
  • Late
    • Even with a well healed wound, increasing pain can be associated from infection seeded hematogenously
  • Latent
    • May have negative intra-op cultures if the bacteria is only on the rods
  • Presentation
    • For late infections, there may only be pain
    • Take a careful history of pain, night pain, contitutional symptoms
    • Rule out any neurological/bowel/bladder
    • Get history of old or reports
    • PMHx and risk factors
  • Labs
    • ESR and CRP are the most sensitive
  • Imaging
    • XR usually not helpful
    • MRI is the most useful but can be limited by post-inflammatory changes
      • Rim enhancing fluid collection is pathognomonic for infection
138
Q

This patient had a discectomy one year ago and now presents with increasing pain. No drainage, wound is healed and no neurological comprimise. ESR and CRP are up. What is the diagnosis? Treatment?

A

Post-op Disc Infection with Rim Enhancing Lesion

  • Presentation
    • Increasing pain
    • complete and AMPLE history
    • other infectious symptoms
  • MRI will show rim enhancing lesion
  • Mangement
    • Blood and urine cultures
    • Needle biopsy for cultures
  • ID consult and IV abx for minimum 12 weeks
    • follow blood work
    • change to PO when responding
  • Indications for surgery
    • Refractor to antibiotics
    • Infection in spinal canal
    • Neurologic deficit
    • Instability
  • Decompression usually done anteriorly, but can be done posteriorly in the lumbar spine
139
Q

Patient comes in 2 weeks following epidural injection and elevated ESR/CRP. Treatment?

A

May not have erythema, redness
Present with increasing pain
ESR/CRP and MRI are usually helpful
Usually respond to antibiotics alone

140
Q

Despite your better judgement, you perform spinal decompression with instrumented fusion in diabetic smoker. They come in 3 months later with a chronic draining sinus. Management?

A
  • ESR, CRP, WBC
  • Urine and blood cultures
  • XR to look for loose hardware
  • CT pre-op to look for fusion
  • ID consult
    • Will need minimum 6 weeks IV antibiotics
    • Follow with serial blood work
  • Aggressive ID, removal of loose graft or loose hardware
    • If there is not yet fusion try to retain hardware
    • Try and treat with antibiotics until you can get the fusion mass to heal, then remove hardware
    • Take deep cultures
    • Close with VAC
  • Multi-organism infections will require repeat debridement every 48-72 hours
    • Take cultures at each I&D
    • Alter or add abx as necessary
  • Plastic surgery consult for soft tissue coverage
  • PT for early ROM and mobilization
141
Q

What is the classification of an infected spinal wound

A
  • Group Anatomic Type/Severity of Infection
  1. Single organism (superficial or deep)
  2. Multiple organism, deep
  3. Multiple organism with myonecrosis
  • Physiological/Host Characteristics
  1. A Normal
  2. B Local or multiple systemic disease (eg, smoking)
  3. C Immunocompromised (Injury Severity Score >18)
142
Q

Describe the transperitoneal approach

A
  • Considerations
    • transperitoneal has a higher risk of retrograde ejaculation (superior hypogastric plexus)
    • if you go from left it’s easier, because easier to mobilize arteries rather than frail vessels
    • Common Iliac branch at L4
  • Incise the rectus and transversalis
  • Find the arcuate line (fasica of the rectus); incise it and use it to retract the peritoneum
  • Iliolumbar vessles branch off the internal iliac and need to be ligated for exposure to L4/5
  • For exposure to L5/S1 the median sacral artery needs to be identified and ligated
143
Q

Describe vertebrobasilar insufficiency

A
  • getting tinnitus, diplopia, nystagmus, vertigo, facial palsy
  • due to occlusion of the vertebral artery
  • Wallenberg syndrome
    • complete brainstem infarct
    • falling to side of lesion
  • Medial medullary syndrome
    • diplopia
    • contralateral arm and leg syndrome
  • Basilar artery syndrome
    • “locked-in”
144
Q

Describe the retroperitoneal approach

A
  • less risk of retrograde ejaculation
  • go from left to avoid liver and veins
  • Lateral decibuitus
  • T12
    • may need to incise the diagphram, leave a cuff of tissue
    • attaches to the vertebrae, xiphoid and lower 6 ribs
  • Incise the transversalis fascia and enter behind the peritoenum
  • be careful of ureter, and genifemoral nerve that sits on psoas
  • disect the psoas off the vertebrae and ligate the segmental vessels
145
Q

Contranindications to C1-C2 translaminar screw of magerl

A

large/medial VA (CT angio)
nonanatomic reduction C1/C2
hypoplastic C2 pars
substatial thoracic kyphosis

146
Q

List 5 surgical options for degenerative spondylolisthesis

A
  • Decompression alone
  • Laminectomy and non-instrumented fusion
  • Decompression posterior instrumented fusion
  • Decompression, PIF and interbody fusion (PLIF, TLIF)
  • MIS decompression
  • MIS decompression and TLIF
  • Dynamic stabilization
147
Q

Characteristics of the ideal bone graft

A
  1. biocompatible
  2. bioremodeled
  3. osteoconductive
  4. osteoinductive
  5. easily acquired
  6. cost effective
148
Q

Risk factors for pseudoarthrosis of posterior fusion

A
  • Nothing to show that PLIF is a good augment to PLF alone
  • Risk factors
    • smoking
    • diabetes is NOT
  • Augments
    • BMP-2
    • autograft
    • allograft mixed with autograft
    • gels not supported
    • demineralized bone matrix not supported
149
Q

What are the components of TLICS

A
  • Injury morphology
    • 1 - compression
      • +1 burst
    • 3 - rotation
    • 4 - distraction
  • Neurological status
    • 2 - nerve root
    • 3 - incomplete SCI
    • 2 - complete SCI
    • 3 - cauda equina
  • Posterior ligamentous complex
    • 2 - suspected
    • 3 - disrupted
150
Q

What are the components of the 3 columns of the spine

A
  • anterior column
    • anterior longitudinal ligament (ALL)
    • anterior 2/3 of vertebral body and annulus
  • middle column
    • posterior longitudinal ligament (PLL)
    • posterior 1/3 of vertebral body and annulus
  • posterior column
    • pedicles
    • lamina
    • facets
    • ligamentum flavum
    • spinous process
    • posterior ligament complex (PLC)
    • instability defined by
151
Q

What makes up the posterior ligamentous complex

A

supraspinous ligament
interspinous ligament
ligamentum flavum
facet capsule

152
Q

Indications for surgical stabilization of a burst fracture

A

injury to the Posterior Ligament Complex (PLC)
progressive kyphosis
> 30°kyphosis (controversial)
> 50% loss of vertebral body height (controversial)
> 50% canal compromise (controversial)

neurological deficiet

Otherwise the best treatment is bracing for pain control

153
Q

Compare anterior vs posterior approach to burst fracture

A
  • Anterior
    • direct decompression
      • better for severe comprimise of canal and comminution
      • better for delay to surgery
    • do not stabilize posterior elements
    • fewer segments to stabilize
    • better correction for deformity >30 deg
  • Posterior
    • easier approach
      • better multiple medical co-mobidities or severe trauma
    • lower non-union
      • more stable fore fracture dislocation
    • indirect decompression
      • ligamentotaxis less effective after 5 days
      • can decompress transpedicular
    • try not to do a laminectomy
    • one level above, one below
154
Q

What are the principles of the anterior thoracolumbar approach

A
  • Side of approach
    • > T11 right (aorta on the left)
      • Lat dorsi/serattus
  • End the rib cage
    • intercostal
    • rib resection - better exposure
  • Diaphragm is at T12
    • leave a cuff to repair to
155
Q

Dangers associated with the transthoracic approach

A
  • Intercostal vessels
    • vulnerable during
      • rib resection when running along undersurface of rib, and
      • exposure of vertebrae within chest
    • avoid injury by entering pleura from above the ribs
  • Lungs
    • avoid injury by using sharp instruments wisely when within chest
    • expand lungs every 30 minutes to prevent microatelectasis
  • Esophagus
    • avoid injury through adequate retraction of esophagus while working on spine
  • Artery of Adamkiewicz
    • travels on left side between T9-L2 in 60% of patients
    • must preserve to prevent spinal cord ischemia
156
Q

Dangers associated with the retroperitoneal approach

A
  • Sympathetic chain
    • lateral aspect of vertebral body
  • Genitofemoral nerve
    • anterior surface of psoas muscle attached to fascia
  • Segmental arteries
    • segmental lumbar arteries and veins
  • aorta
  • Ureter
    • lies between psoas fascia and peritoneum
    • attached more firmly to peritoneum
    • stroke to produce peristalsis to confirm
157
Q

What are the corresponding spinal levels to vascularture of the abdomen

A
  • T12 diaphragm
  • L1 - SMA
  • L2 - renal
  • L4 - common iliac bifurcation
158
Q

Complications associated with anterior approach to the lumbar spine

A
  • left iliac artery most common artery
  • left common iliac vien most common vein
  • ureter tear
  • retrograde ejaculation - presarcal superior hypogastric plexus
  • lumbar sympathetic trunk
  • Nerve root injury
    • over distraction
    • iatrogenic disc hernation
    • posterior protrusion of cage
  • Proximal DVT
  • arterial thrombosis
159
Q

Injury to what structure causes retorgrade ejaculation

A

presarcal superior hypogastric plexus

160
Q

What are the risk factors for fracture of a UBC

A

transverse diameter that is .85% of the diameter of the affected bone and a cyst wall that is ,0.5 mm thick

161
Q

Predictors of lumbar spinal stenosis outcomes

A
  • Poor outcomes
    • Depression
    • cardiovascular comorbidity
    • disorder influencing walking ability
    • scoliosis
  • Good outcomes
    • Better walking ability
    • self-rated health
    • higher income
    • less overall comorbidity
    • pronounced central stenosis
  • Male gender and younger age predicted better postoperative walking ability.
162
Q

Describe a lumbar decompression with discectomy

A
  • Posterior midline incision
  • Divide the fascia and come down onto the spinous processes, and remove the SP of the level of interest
  • Wide pedicle to pedicle decompression for cauda equira
  • unilateral laminectomy is better for single sided pathology and discectomy
  • Use loops or magnification to do a laminotomy
    • preserve 50% of the facets bilateraly
    • preserve at least 5mm of the pars
    • remove to the top and bottom of the facet joint and remove the liagmentum flavum
    • debride any osteophytes
      *
163
Q

Closed reduction facet dislocation

A
  • Awake and co-operative patient
  • consult anesthesia for gentle sedation with benzos and airway management
  • supine with appropriate flouro
  • Application of garden-wells tongs
    • 1cm above the pinna at the equator of the skull
    • tighten until the preset spring is loaded
  • in line traction, starting with 10 lbs, every 20 min with repeat neuro checks and imaging
    • max 70 lbs although this is very controversial
  • Stop if:
    • reduction is achieved
    • when > 1 cm of distraction occurs at site of injury
    • when neurological status of pt deteriorates
    • when maximum amount of weight is applied
  • unilateral facet
    • maintain traction and rotate towards dislocation
  • bilateral
    • maintain traction, palpate the stepoff, push the inferior facet anteriorly, then rotate 40 deg each way to reduce the facets
164
Q

surgical treatment of facet dislocations

A
  • Get MRI pre-op to make sure there’s not a disc
  • Maintain spinal precautions and collar
  • Fiberoptic or glidoscope intubation
  • Sandwich and flip
  • Neuromonitoring
  • Anterior alone
    • unilateral
    • do a discectomy then fuse
    • can use caspar pins from the front for unilateral dislocation
      • cannot reduce bilateral from the front
  • Posterior
    • bilateral or unilateral with no disc
    • one level above and below
  • Anterior then posterior
    • anterior to do a discectomy
    • then put in graft and a kick-out plate
      • allows reduction without having to go back anteiror
    • posteiror to get reduction
      • can try unscrubbed
      • caspar pin distractors
      • intervertebral spreader
165
Q

indications to stop closed reduction of facet dislocation

A

reduction is achieved
when > 1 cm of distraction occurs at site of injury
when neurological status of pt deteriorates
when maximum amount of weight is applied